Söker efter standardmodellen

 

61 frågor/svar hittade

Partiklar [123]

Fråga:
Vi upptäckte häromdagen i standardmodellen att lambda-0 och sigma-0 har samma kvarkuppsättning (uds). Hur är detta möjligt? Vad skiljer dem åt?
/

Svar:
Du har rätt, båda partiklarna är uppbyggda av samma kvarkar. Vad som skiljer dem åt är den inre strukturen. Bl a har kvarkarna precis som elektronen spinn. I sigma-0 har u och d-kvarkarna parallella spinn medan dessa två kvarkar har antiparallella spinn i lambda-0. Skillnaden i inre struktur avspeglar sig sedan i partiklarnas egenskaper som t ex massa.

I partikelfysiken har man en lite annorlunda terminologi än i atom- och kärnfysik. Man säger t ex inte att ett exciterad tillstånd i väteatomen är en ny atom. I partikelfysiken däremot så får det exciterade tillståndet status som en egen partikel.

*

Partiklar [1820]

Fråga:
Hejsan! Jo det är så att jag har börjat tvivla på den "sk" standardmodellen inom partikelfysiken. Jag menar att det finns så mycket som tas för givet här inom fysiken. Allt utgår ju nästan från standardmodellen. Jag anser själv att fysiken(speciellt inom partikelfysiken) har stått och stampat i samma spår i många år nu utan att ha (grovt uttryckt kanske) kommit någonstans. Det jag vill ska hända är att forskare runt om i världen skall vara mer "öppna" för nya möjlighter eller ändringar i standardmodellen och mycket annat i framtiden.Därför att forskarna själva säger ju att dom är osäkra på ditt o datt därför att det inte stämmer med standardmodellen! Och för att få vissa saker att stämma så lägger man t.ex till en konstant. Och efter ca 30 års forskning så kan man fortfarande inte bevisa att kvarken existerar! Jag anser att den inte finns! Därför att standardmodellen har så många fel och brister i sin uppbyggnad att felen överskuggar det som stämmer! Men. Som så många andra så kan även jag ha fel. Men det som är det stora hindret är att forskningen nästan TVINGAS att gå åt ett håll därför att annars skulle alla "bidrag" försvinna. T.ex om man vill undersöka en viss grej och får "avslag" så kan man inget göra, även om de flesta forskare vill det ( och det hoppas jag verkligen!) Det skulle bli för dyrt, eller det ligger i "statens" intresse. Detta är ju helt fel! Forskare kan inte utvecklas så mycket som man skulle kunna! Det får utgå från en modell som är allmänt accepterad och som ingen har satt sig in i VARFÖR den har så mycker brister egentligen. Man skulle kunna säga att den stämmer till en viss grad, men sedan menar jag den kollapsar mer och mer. Men som jag sa förut så är det mycket stor osäkerhet inom partikelfysiken. Det märker man mer om man är en lekman som jag och kritiserar vad ni kommer fram till. Men det kan inte vara lätt. Därför önskar jag att ni fortsätter med det ni håller på med! Det är ju egentligen NI som är grunden till all förändring här i världen! (tänker då mest på den elektroniska utvecklingen) Och min fråga till NI Fysiker är om ni instämmer i mitt resonemang eller om ni til en viss del instämmer? Vad är det som får er att fortsätta med ert arbete när ni stöter på motgångar? Skulle vara väldigt tacksam om ni lämnade ett ärligt svar! David Karlsson
/David K, Gudlavbilderskolan, Sollefteaå

Svar:
Oj, det var många synpunkter. Ett par kommentarer:

Experimentalfysiken sysslar inte med att bevisa saker. Därför kommer aldrig kvarkens existens att kunna bevisas. Experimentalfysiken sysslar med att undersöka om en teori (modell är kanske ett bättre ord) ger förutsägelser, som stämmer med verkligheten. Och det kan vi försäkra, det är experimentalfysikerns högsta dröm, att hitta något som avviker från modellens förutsägelser.

Ur den synpunkten är standardmodellen en mycket framgångsrik modell. Inget experiment har hittat något, som motsäger standardmodellen. Däremot finns andra argument för att det finns en mera fundamental modell bakom. Ett sådant är att det finns så många fria parametrar, över 20 stycken. Det är alltså parametrar, som inte kan härledas, utan måste bestämmas experimentellt.

Om kvarkens existens: För den som arbetar inom fältet är kvarkens realitet ganska påtaglig. Nu gäller det upplevelser! 
/KS

*

[2369]

Fråga:
Jo, jag undrar hur många partiklar som man ännu experimentellt inte kunnat påvisa men som teoretiskt borde existera. Dessutom undrar jag om ni kan berätta något om Higgs-partikeln. Jag vore mycket tacksam för svar.
/Jonny K, Trollhättan

Svar:
Higgs-partikeln är den enda partikeln i standardmodellen som återstår att påvisa. I standardmodellen tänker man sig vakuum fyllt av det så kallade Higgsmediet. Detta medium ger massa åt vissa partiklar t ex de som förmedlar den svaga växelverkan. Detta fungerar på samma sätt som när en foton (ljus) går igenom ett genomskinligt medium t ex glas. Ljusets fart i mediet blir då lägre än farten i tomma rymden och detta gör att fotonen uppför sig som om den hade massa. Higgsmediet ger på liknande sätt massor åt kraftförmedlarna för den svaga växelverkan d v s W och Z-bosonerna. Det kan även bli ett slags tryckvågor i Higgsmediet vilket vi uppfattar som en partikel, Higgspartikeln. Denna har som sagt inte hittats experimentellt ännu, men bör ha en massa på mellan 100 och 1000 gånger protonens massa.
/Lars Gislén

*

Partiklar [5957]

Fråga:
Har de subatomära partiklarna, till exempel elektronen, protonen och neutronen, någon utsträckning? Om ja, vilken? (Hur har man i så fall kommit fram till detta?)

Kird
/Namrum K

Svar:
Av de partiklar du nämner, är det bara elektronen som är en elementarpartikel. Enligt standardmodellen är den strukturlös och punktformig. Det har man bekräftat experimentellt ned till 10-18 m. Protonen och neutronen är sammansatta partiklar, och de har en klar utsträckning, ungefär 10-15 m. Det kan man konstatera genom att sprida andra partiklar mot dem. Sök på kvark i denna databas!
/KS

*

Partiklar [6392]

Fråga:
vad fins inuti en kvark
/peter a, mariaskolan, jula

Svar:
Enligt den så kallade standardmodellen är en kvark strukturlös och punktformig.
/KS

Se även fråga 5957 och fråga 3535

*

Partiklar [6427]

Fråga:
En fråga om antimateria: Det sägs att man i princip skulle kunna byta ut all materia mot antimateria utan att man skulle märka någon skillnad (positiv/negativ laddning är ju en definitions fråga), men vid Big Bang skulle skillnaderna i egenskaperna mellan materia/antimateria haft en stor betydelse. Frågan lyder alltså: Förutom laddning, vilka andra egenskaper skiljer materia mot antimateria?
/Anders Z, Stockholm

Svar:
Vi har varit inne på det här tidigare. Den neutrala K-mesonen är den enda partikel man har hittat, där materia och antimateria uppför sig olika. Det kallas CP-brottet, och skillnaden är mycket liten. Det finns en förklaring till detta som ryms inom standardmodellen. Andra okända effekter kan inte uteslutas. Därför är det mycket angeläget att undersöka effekten hos andra partiklar. Den som ligger närmast är den neutrala D-mesonen, som innehåller en B-kvark. Tre olika acceleratorer är helt inriktade på detta. De kallas B-fabriker (B-factories). Kolla länken!
/KS

Se även fråga 5143 och fråga 1543

1 http://www.cerncourier.com/main/article/40/8/2

*

Partiklar [6711]

Fråga:
Jag skulle vilja ha lite fakta om Standardmodellen. Var ska jag leta?
/K S

Svar:
Kolla sajten nedan! Du kan också söka på standardmodellen och kvark i denna databas, eller slå på elementarpartikel i Nationalencyklopedin .
/KS

1 http://home8.swipnet.se/~w-89875/part/part1.htm

*

Universum-Solen-Planeterna [6721]

Fråga:
Hur beräknar man universums ålder?
/Veckans fråga

Ursprunglig fråga:
Hej! Jag håller på med ett specialarbete om universums ålder, och jag undrar hur de fullständiga beräkningarna för detta ser ut?
/Gustaf J, Gislaveds gymnasium, Gislaved

Svar:
Universums ålder är enligt ett naturvetenskapligt sätt att se, den tid som förflutit sedan big bang. De senaste mätningarna med Planckteleskopet, från 2013, ger vid handen att universum är 13,798±0,037 miljarder år gammalt, enligt standardmodellen för kosmologin, Lambda-CDM-modellen. (Universums_ålder )

Det finns i huvudsak två metoder att bestämma universums ålder: kosmologiska och astrofysikaliska. För en del år sen var det en besvärlig situation. De äldsta stjärnorna tycktes vara 15 miljarder år, medan de kosmologiska beräkningarna gav universums ålder till 10 miljarder år. Idag är diskrepansen i stort sett borta (ungefär 14 miljarder år).

De kosmologiska beräkningarna baseras på universums expansionstakt, som ges av hubblekonstanten (H). Wendy Freedman, en av de främsta specialisterna på området, har skrivit en artikel i Scientific American om saken (mars 1998). I princip är universums ålder proportionell mot 1/H. Fullt så enkelt är det inte. Man måste också veta om H ändras med tiden, och det är modellberoende. Nu pekar två oberoende metoder (baserade på supernovor och den kosmiska mikrovågsstrålnigen) på att expansionen ökar med tiden, och det ger ett högre värde på universums ålder.

Det är inte möjligt här att presentera de fullständiga beräkningarna. Det har skrivits hundratals hyllmeter om saken, men sajterna Chandra will target the age of the Universe och Age of the Universe ger mera information.

Hubbles lag

Edwin Hubble upptäckte i slutet på 1920-talet att galaxerna uppvisade rödförskjutning - ju längre bort galaxen var desto större var rödförskjutningen. Hubble tolkade rödförskjutningen som en rörelse bort från oss (med hastigheten v, se länk 1) och fann en proportionalitet

v = H*d

där d är galaxens avstånd, se nedanstående figur, och H är en konstant, hubblekonstanten. Den rimliga tolkningen av denna observation var att universum expanderar och att universum från början var mycket litet. Detta var början av vad som i dag är den kosmologiska standardmodellen, big bang teorin. Den förste som föreslog att universum börjande som en "uratom" var Georges Lemaître.

Bilden är från Indiana University, länk 2. Se vidare Edwin_Hubble , Hubble's_law , Big_bang och Georges_Lemaitre .

Ett par kommentarer om Hubbles lag

Man kan förstå ett par viktiga aspekter på universums expansion med ett enkelt experiment.

Tag en vanlig, rund ballong och måla små prickar på den med en märkpenna. Prickarna skall representera galaxer. Blås upp ballongen lite grann. Mät avståndet mellan två närliggande prickar och två lite längre från varandra. Rita in förbindelselinjerna mellan de uppmätta prickarna. Låt oss säga det mindre avståndet är 1 cm och det större 3 cm. Blås nu upp ballongen så det mindre avståndet är 2 cm. Vad är då det större avståndet? Det bör vara c:a 6 cm. Om expansionen tog t sekunder så är hastighetena 1/t och 3/t. Vi har alltså att expansionshastigheten är proportionell mot det urspungliga avståndet, vilket är Hubbles lag.

Föreställ dig att du sitter på en prick (galax) på ytan av ballongen. Alla andra prickar rör sig bort från dig med en hastighet som alltså ökar med avståndet. Kan du därav dra slutsatsen att du sitter i centrum? Nej, det kan du inte eftersom du gör precis samma observation från alla prickar på ballongytan. Ytan på en ballong har ju inget centrum! Det är samma sak med universums expansion - vår observation betyder inte att vi befinner oss i universums centrum, något som dessutom är ett omöjligt begrepp för ett oändligt universum.

Förenklad beräkning av universums ålder från H

Om vi antar att hubblekonstanten är H=72 km/s/Mparsek kan vi få en uppskattning av universums ålder som tiden = sträckan/hastigheten = d/v = 1/H. (Lägg märke till att dimensionen av 1/H är tid eftersom det finns längd både i nämnare och täljare.) Vi måste först emellertid konvertera Mparsek till km:

1 ljusår = 3*105[c i km/s]*365.24*24*60*60 = 9.47*1012 km

1 parsek = 3.26 ljusår

1 Mparsek = 3.26*106*9.47*1012 km = 30.8*1018 km

Universums ålder 1/H blir då

(30.8*1018 km)/(72 km/s) = 0.428*1018 s = 0.428*1018/(60*60*24*365.24) år = 14*109 år

I verkligheten är expansionshastigheten inte konstant så man använder sig av lite mer sofistikerade kosmologiska modeller, se fråga 11987 och 18686 .



/KS/lpe

Nyckelord: kosmologi [33]; universums expansion [16]; big bang [37];

1 http://hyperphysics.phy-astr.gsu.edu/hbase/relativ/reldop2.html#c1
2 http://www.indiana.edu/~geol105/images/gaia_chapter_1/edwin_hubble.htm

*

Partiklar [7437]

Fråga:
vilken är den minsta partikel som man känner till?
/hannes k, näsbydalskolan, Täby

Svar:
I standardmodellen betraktas kvarkar och leptoner som punktformiga och strukturlösa, alltså riktiga elementarpartiklar. Experimentellt är det kollat ner till 10-18 m, eller en tusendels protondiameter. Sök efter dessa ord i denna databas!

Med minst skulle man också kunna mena lättast. Då är det säkert elektronneutrinen (en neutral lepton). Massan skulle kunna vara noll, men vissa mätningar kan tolkas som att den inte är det. I varje fall är den mycket lätt, mindre än en miljontedels elektronmassa.
/KS

*

Partiklar [7548]

Fråga:

1) På vilket sätt ger Higgspartikeln uphov till det som vi uppfattar som massa?

2) Om man inte hittar partikeln, så säger man att hela Standardmodellen rasar, vad menar man med det?

3) Hur tror ni att man kommer se Higgs i den nya LHC:n (Large Hydrone Colider), som man håller på att bygga på CERN i Schweiz?

4) Vem uppfann Higgs??Jag antar att det var en man som hette Higgs??..eller?
/Danne F, Katedralskolan, Uppsala

Svar:

1. Sajt 1 försöker förklara det på enkel svenska. Sajt 2 är på engelska.

2. Då måste vi hitta något annan mekanism som genererar partikelmassor.

3. Higgspartikeln väntas mest sönderfalla till 2 b-kvarkar, som vardera vanligen sönderfaller till två strålar av hadroner (starkt växelverkande prtiklar). Man kommer leta efter kollisioner där det finns 4 hadronstrålar.

4. Peter Higgs är engelsman. Han var inte alldeles ensam om detta, så är det för det mesta.
/KS/lpe

Se även fråga 1375

Nyckelord: higgspartikeln [10]; standardmodellen [24];

1 http://courses.physics.kth.se/5A1310/elementar/symmetrier.html
2 http://atlas.web.cern.ch/Atlas/documentation/EDUC/physics9.html

*

Universum-Solen-Planeterna [8357]

Fråga:
Vad är Strängteorier- M-teorin?
/Veckans fråga

Ursprunglig fråga:
Vad är Strängteorier- M-teorin?
/Johanna T, Fyrisskolan, Uppsala

Svar:
Nu berör du saker som verkligen handlar om spetsforkning i teoretisk fysik. Strängteorier kom till för kvantisera gravitationen. Kvantmekanik och allmän relativitetsteori går nämligen inte ihop. Strängteorier arbetar i 10 dimensioner, 9 rumsdimensioner och en tidsdimension. När man 1995 studerade stark koppling i vissa strängteorier vek det ut sig ytterligare en rumsdimension, och den endimensionella strängen blev ett 2-dimensionellt membran i det 11-dimensionella tidsrummet. De 5 etablerade strängteorierna visade sig ha ett visst samband genom det 11-dimensionella tidsrummet. Vad som döljer sig där kallas M-teori. Man kan betrakta de 6 strängteorierna som olika 10-dimensionella projektioner av den 11-dimensionella M-teorin. Det arbetas intensivt på att klara ut vad M-teorin egentligen är. Det mesta är okänt, men många hoppas att här döljer sig svaret på många av universums gåtor. Det finns de som tror det blir möjligt att formulera en teori helt utan fria parametrar. Det skulle vara ett oerhört framsteg. Den så kallade "standardmodellen" har 19 fria parametrar, som alltså måste bestämmas experimentellt

Av de många rumsdimensionerna återstår av någon anledning bara 3. De övriga är inkrökta med sådan liten radie, att de inte märks.

Vill du fördjupa dig i detta kan vi rekommendera: Brian Greene: Ett utsökt universum. Det är en populärvetenskaplig bok, men den är skriven av en expert, och översatt till svenska av en expert (Hans-Uno Bengtsson).

Här är en trevlig websajt som bland annat behandlar strängteori: The Official String Theory Web Site .
/KS/lpe

Nyckelord: standardmodellen [24]; strängteori [7];

*

Partiklar [9598]

Fråga:
Jag har läst lite om standardmodellen, som har intresserat mig, men jag undrar hur de har bestämt vilka partiklar som ska höra till familj 2 resp 3. Familj 1 är ju de partiklar som finns i alla våra atomer men de andra existerar bara i laboratorium som jag har förstått det.
/Marie H, Bromans gymnasium, Harmånger

Svar:
Det här är ett intressant problem. Man tycker kanske att familj 2 och 3 är helt onödiga och ointressanta. Faktum är att utan dessa familjer hade vi inte funnits till, ja universum hade varit helt tomt på vanlig materia.

Man har anlednig att tro att universum till en början var symmetriskt med avseende på materia och antimateria. Vid ett visst tillfälle uppstod en liten asymmetri, och där var familj 2 och 3 inblandade. Sedan förintades (annihilerades) det mesta, det återstår bara 0.0000000001, och det är vad vi består av.

Sedan till din egentliga fråga. Normalt sönderfaller en familj 3 partikel till en familj 2 partikel, som i sin tur söderfaller till en familj 1 partikel. Dessa sönderfall sker med svag växelverkan varför dessa partiklar lever mycket längre än om de hade kunnat sönderfalla med stark växelverkan.

Du kanske förstår att detta resonemang är något förenklat, men vi kan inte vara alltför invecklade här. Se vidare t.ex. Standardmodellen - bilden nedan på det tre familjerna av elementarpartiklar är från denna sajt.



/KS

Se även fråga 1543

Nyckelord: standardmodellen [24];

*

Partiklar [10116]

Fråga:
En fråga om partiklar den här gången:

Atomen är ju mestadels tomrum, den volym som upptas av kärnan är ju nästan försumbar. Hur är förhållandet för själva nukleonerna - dvs hur mycket är tomrum mellan de kvarkar som bygger upp protoner, neutroner, osv?

Har man någon teori om vad kvarkarna är uppbyggda av eller anses dessa vara de minsta partiklar som (hittills) upptäckts?
/Bengt I, Sven Eriksson, Borås

Svar:
Om atomen mest består av tomrum kan verkligen diskuteras. I Bohrs atommodell är det så, men den modellen är övergiven sedan mer än 3 generationer. I den kvantmekaniska beskrivningen av den neutrala väteatomen finns elektronen överallt. Sedan är det en annan sak att atomkärnan upptar en mycket liten del av atomens volym.

I standardmodellen för elementarpartiklar betraktas leptoner (som elektronen) och kvarkar som strukturlösa (punktformiga). Det betyder inte att de är lokaliserade i en punkt i rummet, det förbjuder obestämdhetsrelationen. Det betyder att kvarkarna och gluonerna i en proton befinner sig överallt. Frågan om tomrummet i protonen är alltså meningslös ur kvantmekanisk synpunkt.

Man arbetar med att förena allmänna relativitetsteorin med kvantmekaniken. Dessa teorier kallas supersträngteorier. Här är elementarpartiklarna små vibrerande strängar, alltså inte längre punktformiga. Detta är bara fråga om teorier, alltså inte några experimentella resultat.
/KS

Se även fråga 3652 och fråga 9467

*

Partiklar [10288]

Fråga:
Vad kommer egentligen nukleoners massa ifrån. Hur kan dessa partiklar ha massa. Kommer detta från energin de innehar? väger en elektron 0,0054858 u utan materia eller är det dess innebärande energitillstånd som gör att den väger?
/Nicole d, Lunds gymnasium, Lund

Svar:
En proton innehåller kvarkar, antikvarkar och gluoner. Du förmodar alldeles rätt. Protonens massa består i huvudsak av dessa partiklars energi.

Elektronens massa har man för närvarade ingen förklaring på. Den så kallade "standardmodellen" säger inget om det. Det är en anledning till att man letar efter en djupare, bakomliggande teori.
/KS

*

Kraft-Rörelse [10444]

Fråga:
Vad menas med att genom att låta tex fotoner vakuumpolariseras och växelverka med ett Higgs-fält så känner de ett mostånd och därmed är de massiva-de har massa. Vad har Higgs-partikel med motståndet i vakuum att göra? Är Higgs-partiklarna det som utgör motståndet? Är det faktum att de kan känna motstånd bevis på att de har massa?' En sista fråga: Vad menas med att gravitationen är kvantiserad och varför sägs den vara det?
/Maria E, Åsö, Stockholm

Svar:
Begreppet vakuumpolarisation hör i första hand hemma i kvantelektrodyamiken, och där finns inga Higgsfält. Alltså, det motstånd ljuset känner i vakuum genom vakuumpolarisationen (som bestämmer ljushastigheten) beror inte på något Higgsfält. I varje fall inte på något uppenbart vis.

Higgsmekanismen konstruerades för att förklara varför fotonens tyngre kompisar
( Z0,W+ och W- ) är så tunga. Också de andra elementarpartiklarna
(kvarkar och leptoner) får sina massor på detta vis. Fotonen är masslös just därför att den inte känner av Higgsfälten. Detta enligt den så kallade standardmodellen. Higgsmekanismen är ännu inte experimentellt bekräftad. Kolla sajten (på engelska).

Den väl etablerade gravitationsteorin (allmänna relativitetsteorin) går inte ihop med kvantmekaniken, och det är ett bekymmer. Man jobbar intensivt med att hitta en kvantiserad gravitationsteori, men man är inte framme. Man anar vagt något, som går under beteckningen M-teori.
/KS

Se även fråga 8357 och fråga 9467

1 http://www.sciam.com/askexpert_question.cfm?articleID=000290D2-391C-1C71-84A9809EC588EF21&catID=3&topicID=13

*

Partiklar [10689]

Fråga:
En proton som krockar med en elektron bildar en neutron. Men vad händer ifall en neutron krockar med en elektron?

/Dr Singh
/Manjinder S, Polhemsgymnasiet, Göteborg

Svar:
Enligt den så kallade standardmodellen kan en proton och en elektron inte bilda en neutron. Det skulle ändra det så kallade leptontalet (ungefär: antalet elektroner och neutriner).

En tillåten reaktion skulle vara att det bildas en neutron och en neutrin. Det sker i så fall med svag växelverkan, och det är så sällsynt att man kan glömma bort det i alla praktiska sammanhang.

Med en elektron och en neutron händer ingenting (vid låga energier).
/KS

*

Partiklar [11101]

Fråga:
Kan materia förekomma i hur små portioner som helst? En elektron har jag aldrig sett kunna delas, men kvarkar, kan de delas? Neutriner har ev massa, om än väldigt liten; kan många neutriner slås samman till en större partikel? (laddningsproblem skulle det ju inte bli!) Kan fotoner ha massa, om än liten? Kan det förekomma kraftverkan mellan fotoner? neutriner? Är även neutriner partikel-våg -dualistiska till sin karaktär?
/Thomas Å, Märstagymnasiet, Märsta

Svar:
I standardmodellen är elementatpartiklarna (kvarkar och leptoner) punktformiga och odelbara. Att neutriner skulle kunna slå sig ihop till större aggregat finns ingen anledning att tro. Den experimentella övre gränsen för fotonens massa är 0.0000000000000006 eV/c2. Jämför med elektronens massa, som är 511000 eV. Växelverkan kan förekomma mellan fotoner, men inte genom direkta processer. Växelverkan mellan neutriner är i princip möjligt, men det har bara varit av betydelse när universum var mycket ungt med mycket hög densitet. All materia har partikel- vågdualitet.
/KS

*

Partiklar [11164]

Fråga:
Vad talar emot att även elektronen i sig har atomstruktur?
/göran p, bränkeskolan, linköping

Svar:
I den så kallade standardmodellen betraktas elektronen som en punktformig partikel. I de försök som görs att utvidga standardmodellen (supersträngteorier) är elektronen inte längre punktformig, utan en vibrerande sträng. Men den är oerhört liten.
/KS

Se även fråga 10116 och fråga 7985

*

Partiklar [11709]

Fråga:
Hej! Jag håller på med ett projektarbete i partikelfysik och undrar nu ifall ni har några tips på hemsidor där man kan hämta utförliga tabeller som har med standardmodellen att göra.

Med vänliga hälsningar

Jimmy Kungsman
/Jimmy K, Martin Koch, Hedemora

Svar:
Något vettigt på svenska har vi inte hittat. Här är 2 förslag. Där finns länkar för fler detaljer.
/KS

1 http://ltp.web.psi.ch/
2 http://www2.slac.stanford.edu/vvc/theory/model.html

*

Partiklar [11879]

Fråga:
Hej! Av svaret på min förra fråga, partiklar 11846, blir jag förvånad; är verkligen en neutrino en oladdad elektron?? Neutrinernas massa är ju omdebatterad och följden blir ju att det är elektronens laddning som är massan medan den oladdade massan ev inte finns ! Finns det någon bra bok i ämnet, gärna på svenska, som vi på gymnasiet skulle kunna läsa om hur det hänger ihop i?
/Thomas Å, Märstagymnasiet, Märsta

Svar:
At säga att "neutrinen är en oladdad elektron" är kanske något slarvigt, men det ligger faktiskt en del i det. Dessa partiklar hör till en grupp som kallas leptoner. Det finns tre laddade leptoner: elektronen, myonen och tauonen. De har sin motsvarande oladdade partner: elektronneutrinen, myonneutrinen och tauonneutrinen. I den så kallade standardmodellen är dessa neutriner olika partiklar.

På senare år har det visats att standardmodellen inte kan vara korrekt på denna punkt. De olika neutrinerna kan övergå i varandra (neutrinooscillation). Det innebär att neutrinerna inte kan vara masslösa, som antas i standardmodellen. Dessa resultat är så färska (detta skrives april 2003) att de knappast finns i någon bok. Vi får hänvisa till ett par sajter.
/KS

1 http://neutrino.lbl.gov/
2 http://www.physics.carleton.ca/research/sno/

*

Blandat [15616]

Fråga:
Hej! I ett "papper" av Göran Jarlskog, En värld av nästan ingenting, står det: En av hörnstenarna i fysikens värld är lagen om energins bevarande. Jag undrar vilka de andra hörnstenarna anses vara. Man behöver ju inte ha rätvinkliga hörn, så de kan vara fler än fyra. Men vilka är de? Och finns de listade? På svenska?
/Thomas Å, Arlandagymnasiet, Märsta

Svar:
I klassisk fysik har vi bevarandelagarna för

energi
rörelsemängd
rörelsemängdsmoment
laddning

I kvantmekaniken och standardmodellen tillkommer ett par absoluta, men många av de övriga är inte absoluta utan har undantag. Se vidare konserveringslagar i Nationalencyklopedin , Conservation_law och länk 1 (de senare två tyvärr endast stubbar vilket är Wikipedia-slang för påbörjade artiklar).
/Peter E

1 http://sv.wikipedia.org/wiki/Konserveringslag

*

Universum-Solen-Planeterna [13242]

Fråga:
Hur fungerar Big Bang?
/Veckans fråga

Ursprunglig fråga:
Enligt teorin skapades universum i en sk "big bang". Det måste ju rimligtvis betyda: Ur ingenting skapades allting! Finns det någon teori som ger en god förklaring om hur detta fungerar!?
/Sven E, Furuhedsskolan, Kalix

Svar:
Big Bang (eller Stora smällen), är standardteorin om universums uppkomst. Enligt denna teori skapades universum och rumtiden för ca 13,7 miljarder år sedan, då universum började expandera från att ha varit koncentrerat i en punkt. Termen myntades av astronomen Fred Hoyle (som var motståndare till den) under en radiointervju i BBC den 28 mars 1949. Termen i sig är dock missvisande då det inte handlar om en explosion av materia i en tom rymd utan istället om en expansion av rummet självt som materian befinner sig i. (Från Wikipedia Big_Bang ).

Man är ganska överens om beskrivningen hur Big Bang gick till. Vad som fanns före Big Bang och vad som finns utanför vårt universum vet man inget om även om det finns spekulationer. Speciellt vet vi inget om hur universum skapades (dvs vad som orsakade Big Bang). Vi kan med våra observationer bara "famla lite i kanterna", ungefär som en blind utforskar ett djupt hål genom att känna längs kanten.

Fysiken för det tidiga universum är i gränslandet mellan kosmologi (vetenskapen som behandlar universums uppkomst och utveckling) och filosofi eftersom vi ännu inte har en fullständig teori för hur alla de fyra grundläggande krafterna förenas. Det finns därför inget som länkar vad som hände i det tidiga universum (före Planck-tiden 10-43 s) med vad vi kan observera i dag. Detta gör sådana spekulationer till mer filosofi än vetenskap.

Supersträng-teorin hävdar att universum hade 10 dimensioner under Planck-eran. Dessa övergår 4 dimensioner efter Planck-eran, och de 6 dimensionerna är fortfarande förkrympta och märks alltså inte. Under Planck-eran kan man beskriva universum som ett kvant-skum med 10 dimensioner och som innehåller Planck-längd stora svarta hål som skapades och försvann utan orsak och verkan. Med andra ord: försök att inte tänka på denna eran!

C:a 10-35 sekunder efter Big Bang var det en mycket snabb expansion av universum. Detta fenomen kallas inflation. Observera att denna inflation skedde med överljushastighet. Detta är inget brott mot den speciella relativitetsteorin eftersom den var en expansion av universum självt och inte materian. Vårt synliga universum är då en bubbla - i nedanstående bild den gula bubblan markerad "us". De andra bubblorna är då i någon mening inte reella eftersom de är utanför vår horisont och vi kommer aldrig att kunna kommunicera med dem. Observera alltså att HELA rymden expanderar- även avståndet mellan bubblorna. Detta betyder att två bubblor som inte är i kontakt med varandra vid en viss tidpunkt aldrig kommer att bli det!

Inflationen orsakades av att symmetrin mellan den starka kärnkraften (färgkraften) och den elektrosvaga växelverkan bröts. Detta orsakade en "fasövergång" som gav energi till att driva den snabba expansionen.

Vissa teorier säger att hela vårt universum är ett svart hål med energin noll, se Zero-energy_universe . Eftersom vi aldrig kan kontrollera detta är det en teori som är lika mycket värd som andra. Fenomen som inte kan mätas brukar inte klassificeras som vetenskap. Men det kan ändå vara roligt att filosofera om det ! I artikeln nedan (länk 1) finns en ljudfil som visar hur Big Bang lät. Länk 2 ger mer ganska elementär information på svenska om big bang. Se även övriga frågor big bang och Wikipedia-artikeln Big_bang . Den kände populärvetenskaplige författaren John Gribbin har intressanta funderingar om universum i John Gribbin's home page (Introduction to Cosmology). James Schombert v6.2 är en guldgruva med föreläsningar bland annat om kosmologi. TalkOrigins Evidence for the Big Bang är en omfattande och ganska avancerad FAQ.

Tidslinje för Big Bang


Tid           Temperatur  Händelse
0                         Big Bang
10-43 s                    Planck-tiden, kända naturlagar gäller
10-35 s                    Inflation
300 s                     Bildande av 4He
380000 år      3000 K     Kosmisk bakgrundsstrålning 
13.7*109 år      3 K       Nu

Låt mig avslutningsvis försöka besvara några vanliga frågor om Big Bang.

Vem hittade på big bang?
Aleksandr Fridman och Georges Lemaître föreslog redan på 20-talet att universum uppkommit genom att en "uratom" expoderade. Den ukrainske fysikern George Gamow (George_Gamow George Gamow (George_Gamow förutsade 1948 även att det överallt i universum skulle finnas mikrovågsstrålning med temperaturen c:a 5 K. Uttrycket big bang var från början en nedlåtande beteckning som en motståndare Fred Hoyle (som föreslagit den s.k. steady state teorin, Steady_State_theory ) hittade på.

Vad hände före Big Bang?
Frågan är, som antytts ovan, meningslös eller utan innehåll. Det är som att fråga: vad finns norr om nordpolen? Före Big Bang fanns ingen tid, och man kan därför inte tala om vad som hände.

Vilka bevis finns det för Big Bang teorin?
De viktigaste är

Anses Big Bang-teorin numera vara så etablerad att man inte kan ifrågasätta den?
I stora drag, ja. Alternativet, Fred Hoyles Steady State teori, får nog anses överspelad. Dels var den lösningen på problemet att universum tycktes vara yngre än vissa gamla stjärnhopar. Detta är löst i dag genom att avståndsskalan har ändrats mycket. Dels förklarar Steady State teorin inte den kosmiska bakgrundsstrålningen och heliumförekomsten i gamla stjärnor, något som Big Bang teorin gör elegant. Detaljerna i Big Bang teorin kan säkert komma att revideras med nya observationer. Vi skall också komma ihåg att en fysikalisk teori beskriver vad vi kan observera, och säger inget om t.ex. varför universum började expandera eller vad som händer utanför vår händelsehorisont (så långt vi teoretiskt kan se, dvs i princip ljushastigheten*universums ålder).

Om all materia, ljus som mörk, varit samlad i en punkt, singularitet1 eller uratom, måste väl gravitationen ha varit oändligt stor, åtminstone ögonblicket efter att expansionen startat och fysikens lagar börjat gälla. Då är det svårt att förstå hur expansionen alls kunde ske, hur den kunde övervinna den ofattbara gravitationen, när inte ens gravitationen i ett s.k. svart hål tillåter något att slippa ut.
Ja, det är svårt att förstå. Fysiken kan ibland med trick hantera sådana här singulariteter (oändligheter), men innan 10-43 sekunder efter Big Bang kan vi i dag inte ge en bra beskrivning. Ett trick som används t.ex. för svarta hål är kosmisk censur. Detta betyder att singulariteten existerar endast matematiskt och inte som en fysisk verklighet som vi kan observera eller mäta. Svarta hål omger sig nämligen av en händelsehorisont som gömmer ("censurerar") singulariteten. Se vidare fråga 14367.

Vidare är det förbryllande att man kan se universum strax efter big bang när man tittar riktigt långt bort. Att man ser bakåt i tiden förstår jag gott, men det ljus som skickades iväg under den första tiden borde väl sedan länge ha passerat oss och fly bort ifrån oss med ljusets hastighet. Ser vi det ljuset "på ryggen" och i rakt motsatt riktning mot det ställe i universum där det hela började? Hur ser det i så fall ut när vi riktar våra teleskop ditåt, mot expansionens centrum?
Vår del av universum (det synliga universum) är enligt standardmodellen bara en liten den av vad som skapades vid inflationen. Varje liten bubbla i figuren nedan är ett universum, men de är alla ekvivalenta och inget innehåller expansionens centrum. Detta är svårt att förstå om man går ända tillbaka till tiden noll, men det kan vi alltså inte göra. Vad vi ser om vi går så långt bort som möjligt (13.7 miljarder år) är eldklotet som hade en temperatur på 3000 K, men som nu pga expansionen har en temperatur på 3 K.

Man kan fråga sig varför universum är så homogent (den kosmologiska principen, universum har samma egenskaper i alla riktningar). Om man tittar åt ett håll 14 miljarder ljusår bort och i motsatt riktning på samma avstånd, så har båda områdena nästan exakt samma temperatur. Eftersom de inte kan ha stått i kontakt med varandra (avståndet är 28 miljader ljusår så ljuset kan inte ha hunnit gå hela vägen mellan dem) kan man tycka detta är konstigt. Anledningen är inflationen. Detta var ett av skälen till att man införde inflationen. Före denna snabba exansionen var de två områdena så nära varandra att de kunde vara i termisk jämvikt.

En konstighet med universums expansion är det faktum att galaxer kan kollidera trots att rymden mellan dem hela tiden utvidgar sig. Återigen, om de dras till varandra av gravitationen så borde väl gravitationen ha förhindrat att de först avlägsnade sig från varandra.
Mja, man får inte se det så. Det är rymden mellan galaxerna som expanderar. Galaxhopar (grupper av galaxer) är bundna med tyngdkraften och galaxernas rörelse inbördes i hopen bestäms av gravitationen och inte expansionen. Vår granngalax Andromedagalaxen, som befinner sig på c:a 2.5 miljoner ljusårs avstånd, rör sig faktiskt mot vår vintergata i stället för att avlägsna sig som de flesta andra galaxer gör.

Sammanfattning av de viktigaste bevisen för Big Bang

1 Rödförskjutning: Galaxernas spektra är rödförskjutna med ett belopp som är proportionellt mot avståndet: Hubbles lag v=d*H, där v är hastigheten, d är avståndet och H är hubblekonstanten.

2 He förekomst: Förekomsten av He i de äldsta stjärnorna är 25% vilket är precis vad Big Bang modellen förutsäger, se fråga 13117 .

3 Kosmiska bakgrundsstrålningen: Mikrovågsstrålningen med en temperatur av 3K härrör från c:a 400000 år efter Big Bang då universum blev transparent genom att H/He kärnorna rekombinerade med elektroner.

Se vidare Big_Bang och på engelska Big_Bang_Theory

___________________________________________________________
1 Singularitet. I matematiken definieras singularitet som en odefinierad punkt hos kurva, yta eller funktion. I kosmologi definieras singularitet som en punkt i rumtiden i vilken rumtidskrökningen är oändlig (svart hål).



/Peter E

Se även fråga 13117 och fråga 14367

Nyckelord: big bang [37]; inflation [7]; kosmologi [33];

1 http://www.newscientist.com/article.ns?id=dn4320
2 http://kasper.pixe.lth.se/NuclearPhysics/slideShow/nobel2006/nobel2006_files/frame.htm

*

Materiens innersta-Atomer-Kärnor [13435]

Fråga:
Det verkar konstigt att en taupartikel är en lepton men kan faller sönder till andra partiklar. Leptoner är elementarpartiklar. Som de skriver på "hands-on cern": "de är inte sammansatta av ännu mindre beståndsdelar."
/Aron P, IT-gymnasiet, uppsala

Svar:
Det kan tyckas konstigt, men så är det! Det är inget unikt med tau-partikeln, även kvarkar kan förvandlas till andra kvarkar. Det vanliga betasönderfallet hos en neutron ses i standardmodellen (se elementarpartikel i Nationalencyklopedin ) som nerkvarkens sönderfall till en uppkvark med utsändande av en W-. W-bosonen sönderfaller sedan i en elektron och en antineutrino. Så enligt nuvarande teori behövs inte en inre struktur för att en partikel skall kunna sönderfalla. Det kan emellertid tänkas att denna inre struktur finns, och att vi inte är medvetna om den ännu.
/Peter E

*

Materiens innersta-Atomer-Kärnor [13562]

Fråga:
Hejsan. Jag har på flera ställen stött på ordet flytande vacuum. Bland annat i artikeln nedan.

Flera mer fysiskt skolade peroner jag pratat med har avfärdat flytande vacuum som någont som inte kan finnas. Hur fungerar det egentligen? Kan det finnas? Och hur ska jag förklara det för dem för att de ska förstå att det faktiskt finns?
/Kim D, Lernia, Västerås

Svar:
Det finns en referens i artikeln till en annan artikel som är publicerad. I den artikeln har man enligt abstract studerat 'liquid model of the QCD vacuum'. Jag förstår inte mycket av artikeln, men man skall vara försiktig med att tolka uttryck i fysik och speciellt partikelfysik för bokstavligt. Det är tydligen något i standardmodellen (se länken) som säger att vakuum kan ha egenskaper på något sätt analoga med en vätska. Tänk även på att standardmodellen säger att kvarkarna har färger (egentligen färgladdningar), vilket man inte heller få ta alltför bokstavligt.
/Peter E

1 http://www.scielo.br/scielo.php?script=sci_arttext&pid=S0103-97332001000100013&lng=es&nrm=iso

Avancerad sökning på 'standardmodellen' i denna databas

*

Materiens innersta-Atomer-Kärnor [13796]

Fråga:
Hur många olika sorters neutriner finns det?
/Veckans fråga

Ursprunglig fråga:
Hej igen, här är några funderingar som jag fick när jag skrev ett skolarbete i fysiken.

1. Som jag har fattat det så består t.ex. en proton av 3 kvarkar (2 uppkvarkar och 1 nedkvark) men jag har märkt att man säger att den också består av 5% antikvarkar. vad menar man med det, en kvark kan väl inte vara delad på något sätt?

2. Jag läste också att man letar efter Neutriner men är man inte redan säker på att det finns en neutrin i varje partikel familj (1=elektronneutrinon, 2=myonneutrinon och 3=tauneutrinon). Så letar man efter nya neutriner eller vill man bara hitta mer av de som redan är upptäckta för expriment?
/Karl J, Hjärteskolan, Trosa

Svar:
1. Kvarkarna rör sig inne i nukleonerna med mycket höga energier. När de kolliderar bildas kortlivade kvark/antikvark-par. Så man kan alltså säga att nukleonerna till en del består av antikvarkar. Om man säger att de till 5% består av antikvarkar så menar man att under 5% av tiden består nukleonen av tre kvarkar plus ett kvark-antikvark par.

2. Jag vet inte om man kan säga att man letar nya neutrinotyper men däremot neutriner från olika källor. Enligt standardmodellen och mycket detaljerade mätningar av vidden hos Z-partikeln från CERN är det ganska klart att det finns tre typer av neutriner. Figuren nedan visar mätresultatet. Det är klart att 2 och 4 typer av neutriner är inkonsistent med data.

Vad man också är ute efter är att bekräfta att neutrinerna har en vilomassa som är skild från noll. I så fall skulle man kunna få s.k. neutrino-oscillationer, dvs att en neutrino av en typ kan förvandlas till en annan typ. Se vidare Neutrino_oscillation .

Se vidare Standardmodellen och Standard_Model .


Nyckelord: standardmodellen [24]; kvark [12];

*

Materiens innersta-Atomer-Kärnor [14074]

Fråga:
Hej, jag har några funderingar om partiklars spinn här som jag skulle vilja ha svar på.. 1. man säger att t ex fotonen har spinn 1 så om vi snurrar den ett helt varv så "uppfattar" den det också som ett helt varv. men hur vet man det? den man kan väl knappast se den eller? 2.finns det någon lista där det står vilka elementar partiklar som har vilken spinn?
/Karl J, Slättmarksskolan, Trosa

Svar:
1. Man skall inte tolka spinn för bokstavligt. Det är en kvantmekanisk storhet.

2. Alla riktiga kända elementarpariklar enligt standardmodellen (länk 1) har spinn 1/2, så det behövs ingen tabell. Sedan finns det kraftförmedlarpartiklarna med heltaligt spinn och naturligtvis sammansatta partiklar med alla möjliga spinn.
/Peter E

1 http://fragelada.fysik.org/index.asp?keyword=standardmodellen

*

Materiens innersta-Atomer-Kärnor [14224]

Fråga:
Hej, vad menar man när man talar om olika frihetsgrader angående higgsmekanismen? Tack
/Karl L, Bergala, Tynum

Svar:
Hej Karl! Jag är nu på Fysikdagarna i Umeå - ett arrangemang både för universitetslärare/forskare och för gymnasielärare.

Din fråga är inte lätt att besvara - trots att jag hade tillfälle att fråga en partikelfysiker! Teorin om Higgs (en del av den sk Standardmodellen, se nedanstående länk) är ganska abstrakt!

Det bästa jag kan säga är att de fyra frihetsgraderna motsvarar de intermediära bosonernas (W+, W- och Z0) massor samt higgspartikelns massa.

Övriga partiklar (leptoner, kvarkar) får sin massa på annat sätt.
/Peter E

Nyckelord: standardmodellen [24];

*

Blandat [14232]

Fråga:
Jag har hört att fysiken är den egentliga naturvetenskapen och all naturvetenskap egentligen är fysik. Är det så?
/Veckans fråga

Ursprunglig fråga:
Jag har hört att fysiken är den egentliga naturvetenskapen och all naturvetenskap egentligen är fysik. Kemi och bilogi skulle alltså vara delar av fysiken. Min lärare säger att det är ur fysiken som kemin kom och ur kemin kom biologin? Vad menas med detta? jag förstår inte riktigt vad min lärare menar när hon säger detta och hon vill inte förklara det mer ingående. mycket tacksam för svar
/Anna L, Gränbyskolan, Uppsala

Svar:
Anna och även Tanja som ställt en liknande fråga!

Att definiera fysik är inte lätt - definitionen beror på sammanhanget. Om man t.ex. menar skolämnet fysik så är det emellertid lätt: fysik är det som ingår i kursplanen för fysikämnet. Om man menar forskningsämnet fysik är det de områden man forskar om på fysiska institutioner vid universiteten. Den preliminära kursplanen för fysik i det nya gymnasiet GY-07 inleds med följande tre stycken:

Ämnet Fysik syftar till fördjupad förståelse för hur fenomen i den egna vardagen och i universum kan förklaras med fysikaliska modeller. Ämnet Fysik syftar också till ett vidgat naturvetenskapligt perspektiv och en modern naturvetenskaplig världsbild. Utbildningen syftar till att öka intresset för ämnet och för fortsatta studier i fysik, matematik, andra naturvetenskapliga ämnen och teknik.

Fysikämnet omfattar allt ifrån det allra minsta, mikrokosmos, till det allra största, makrokosmos. Utbildningen i ämnet syftar till att ge eleven en inblick i olika områden inom fysiken samt dess olika tillämpningar inom vardag, samhälle, industri och forskning.

I all naturvetenskap sker utveckling i samspel mellan experiment och teori. Teorier och modeller är mänskliga tankekonstruktioner som ständigt utvecklas och påverkar människans världsbild. Utbildningen i Fysik syftar därför till ökad kunskap om fysikens arbetsmetoder samt om hur fysikens kunskapsområden utvecklas i samspelet mellan fysik, andra naturvetenskapliga ämnen och matematik.

En alternativ definition är: fysik är den vetenskap som beskriver materia, energi och krafter. Då fysiken är en vetenskap tillämpas den vetenskapliga metoden (se vetenskaplig metod ), med uppställande av hypoteser som antingen förkastas eller antas på grundval av experiment och observationer.

Man skiljer även på klassisk fysik (allt före 1900, t.ex. mekanik, elektromagnetism) och modern fysik (t.ex. relativitetsteori, kvantmekanik, elementarpartikelfysik). Sedan har man även grundläggande fysik (forskning inom fysik som motiveras av vår vilja att förstå naturen) och tillämpad fysik (tekniska, medicinska, mm tillämpningar av fysik).

Eftersom modern fysik omfattar vetande från i stort sett hela 1900-talet, kan det vara på sin plats att använda begreppet nutida fysik som skulle innefatta partikelfysik (standardmodellen ), kosmologi (big bang ), plasmafysik och fusion (fusion ) och kärnfysik, se länk 1 för detaljer om Contemporary Physics Education Project (CPEP).

Historiskt var det så att fysik var all naturvetenskap. Efter hand som kunskapen ökade, hade man behov av specialisering. Efter hand frigjordes biologi, geologi, kemi, m.fl. och blev egna ämnen. Vad din lärare antagligen menar är att t.ex. biologin bestäms helt och hållet av arvet genom DNA-molekylen. De lagar som styr DNAs egenskaper är fysik (kvantmekanik). DNA-molekylernas egenskaper är emellertid så komplicerade att vi inte kommer särskilt långt i förståelsen med kvantmekanik. Man måsta använda andra metoder för att komma framåt, och då är det praktiskt att ge ämnet en egen beteckning.

Historiskt har biologin utvecklats helt oberoende av fysiken: man studerade och klassificerade organismer (Linné, 1700-talet), och efter hand förstod man arv och utveckling (Mendel, Darwin, 1800-talet) och DNA upptäcktes (Watson, Crick, Wilkins och Rosalind Franklin, 1950-talet, se länk 2). Det var egentligen först då kopplingen till fysik återuppstod i form av t.ex. biofysik. Se vidare fråga 13720 .

Även det vi i dag kallar fysik har delats upp i separata ämnen: i början av 1900-talet kallades allt som hade att göra med atomer för atomfysik. Efter hand som kunskaperna ökade frigjordes kärnfysik (som beskrev atomkärnan, atomfysik beskrev bara elektronerna kring kärnan). Efter 1950 frigjordes sedan elementarpartikelfysiken från kärnfysik och blev ett eget ämne som behandlar elementarpartiklar och deras växelverkningar.

För att visa hur krånglig och godtycklig uppdelningen är kan jag säga att delar av kärnfysik i USA klassificeras som kemi (Nuclear Chemistry).

Mer om vad fysik är och vad det är bra för finns under nedanstående länkar och här: fysik, nytta med . Jag beklagar om svaret inte är lätt att förstå, det går nog tyvärr inte att förklara det enklare.

Texten på tröjan nedan antyder att fysiken är den ultimata vetenskapen .



/Peter E

Nyckelord: fysik [10]; *biologi [20]; vetenskaplig metod [18]; kursplan [3];

1 http://www.cpepweb.org/
2 http://www.chemheritage.org/discover/online-resources/chemistry-in-history/themes/biomolecules/dna/watson-crick-wilkins-franklin.aspx

*

Blandat, Energi, Materiens innersta-Atomer-Kärnor [14237]

Fråga:
Randell Mills och pseudovetenskap
/Veckans fråga

Ursprunglig fråga:
En amerikan som heter Randell Mills har tydligen uppfunnit ett system för att få billig och miljövänlig energi från väte. Jag skulle vilja veta mer om detta!
/Sven E, Stockholm

Svar:
Det du frågar om är nog det mest avancerade exemplet på pseudovetenskap och bluff och båg i hela vetenskapshistorien! Nationalencyklopedin säger om pseudovetenskap: mystisk eller spekulativ forskning som inte är accepterad av vetenskapssamhället, t.ex. alkemi, parapsykologi och astrologi. Fallet Mills påminner en hel del om Kall fusion (se fråga 2409 ), men det är ett mycket mer avancerat bedrägeri.

Länken HYDROGEN IS POTENTIAL NEW ENERGY SOURCE beskriver vad idén är. I korthet går det ut på att kvantmekaniken är fel och att det tillstånd vi kallar grundtillståndet i väte inte är det lägsta tillståndet. Det finns enligt Mills flera mycket lägre liggande tillstånd, och man kan genom katalys med kalium få elektronen att gå till dessa lägre tillstånd. Väte i de lägre tillstånden kallar han hydrino. Man skulle då kunna utvinna c:a 40 till flera hundra eV energi från varje atom. Energiutvecklingen ger upphov till en plasma, vilket med vad som kallas en gyrotron kan transformeras till mikrovågor som i sin tur kan generera elektricitet.

Det är uppenbart att om allt detta vore sant skulle man t.ex. kunna köra bilar med vatten som bränsle! Uppfinningen skulle representera ett enormt ekonomiskt och miljömässigt värde. Mills har bildat ett bolag med en mycket professionell webbsajt BlackLight Power, Inc. , och investerare har satsat mycket pengar. Bolaget säljer rättigheter till Mills uppfinning. Bolaget har en fin anläggning i New Jersey, men det tycks bara bestå av direktörer .

Blacklight_Power är en balanserad sammanfattning av Mills idéer och patent. Det är ingen överdrift att säga att etablerade fysiker är måttligt imponerade, och de som är positiva tycks vara direktörer i Mills bolag.

Mills har även skrivit en bok The Grand Unified Theory of Classical Quantum Mechanics med helt nya teorier vad gäller fysiken. Boken (på 1800 sidor i 3 volymer!) ser mycket vederhäftig ut och måste vara resultatet av mycket arbete. Mycket i boken är korrekta textbokskunskaper och en del är fullständig rappakalja. Bland detta finns helt nya vinklingar och teorier, bland annat de nya lägre liggande tillstånden i väte.

En annan intressant sak är att Mills "härleder" förhållanden mellan elementarpartiklarnas massor som funktion av finstrukturkonstanten (en dimensionslös konstant med värdet 1/137.03599911 som förekommer i elektromagnetiska teorin). Bara detta, om det vore korrekt, skulle ge nobelpriset direkt eftersom Standardmodellen för elementarpartiklar och kraftverkningar inte ger några värden på dessa.

____________________

I detta sammanhang kan det vara på sin plats att diskutera vad naturvetenskap är och vad som å andra sidan är pseudovetenskap.

Naturvetenskap - vetenskaplig metod

Ett naturvetenskapligt arbetssätt är ett ständigt samspel mellan teoribyggande och observationer:

TEORI/MODELL <<--->> OBSERVATIONER/EXPERIMENT

I övrigt är följande punkter viktiga för vetenskaplig metod:

  • Alla resutat skall rapporteras i öppna tidskrifter
  • Resultat skall vara testbara och reproducerbara
  • Man får inte godtyckligt välja resultat som "passar"
  • Enklast möjliga beskrivning som inte strider mot tidigare observationer föredras
  • Acceptans för nya idéer – villighet att ompröva gamla teorier
  • Sakargument, ej status och "det är skrivet", bestämmer trovärdighet

Se även ett par frågor som behandlar detta under vetenskaplig metod .

Vad är inte vetenskap?

Vetenskap innefattar alltså det som är mätbart och testbart. Resultat av experiment skall vara reproducerbara. Pseudovetenskap å andra sidan karakteriseras av

  • Data är ofta anekdotiska
    - En vän jag litar på sa att han såg varelser stiga ut ur UFOt
  • Uttalanden är ofta mycket kategoriska eller mycket vaga
    - På vetenskapens nuvarande stadium kan vi säga att...
  • Brist på andra förklaringar
    - Att vi inte kan "förklara" ett fenomen betyder inte att det är övernaturligt
  • Slumpmässiga sammanträffanden är möjliga
    - Bara för att två händelser sker samtidigt behöver inte betyda att de är beroende av varandra
  • Referenser är vaga - auktoriteter används ofta
    - Den välkände Professor Bloggs vid CalTech säger att... I vetenskapen ger man en referens som kan kontrolleras. Det är en persons arbete (normalt publikation) som skall bedömas, inte personens tillförlitlighet
  • Vetenskapligt språk används ofta
    - Bitvis kan fakta vara korrekta – men de är ofta triviala eller irellevanta fakta från läroböcker
  • Hänvisningar till religiösa skrifter
    - Saknar bevisvärde – anektotiska och innehåller ofta symbolik som inte får övertolkas

Observera att det är skillnad på pseudovetenskap, där villfarelsen är avsiktlig, och dålig vetenskap, som i bästa fall kan vara ett oavsiktligt misstag.

Skepticism/källkritik i webbsökningar

Det finns t.ex. på internet väldigt mycket bra information, men också mycket skräp och pseudovetenskap. Några tips för att bedöma information:

  • Leta efter oberoende bekräftelse av fakta
  • Var öppen för olika åsikter, men använd bara bevis som kan bekräftas
  • Studera olika hypoteser – skaffa dig inte en favoritteori som utesluter alla andra
  • Kvantifiera där det är möjligt - "vad som är vagt och kvalitativt är öppet till många förklaringar"
  • Använd "Occam’s Razor": börja med den enklaste förklaringen
  • Är ett uttalande öppet för experimentell bekräftelse? Om inte, kan det vara intressant att diskutera, men validiteten kan aldrig bekräftas
  • Är urspungspersonen knuten till en reputabel institution? Reputabel institution är dock varken nödvändigt eller tillräckligt villkor för tillförlitlighet! (Einstein var anställd vid en patentbyrå i Bern när han pubicerade sina första papper.)
  • Ligger sidan under en officiell website från en respektabel institution (renomerat universitet eller forskningsinstitution)
  • Är sidan från en publikation i en renomerad tidskrift så kan man oftast lita på uppgifterna
  • Icke granskade open access system (t.ex. arXiv, länk 2) kan innehålla allt från nobelpris-forskning till pseudovetenskap, så här gäller det att vara försiktig
  • Är ursprungspersonen expert på ämnet? Nobelpristagare och professorer missbrukar tyvärr ibland sin status för uttalanden i helt andra ämnen!
  • Var misstänksam om författaren utan mycket starka argument angriper grundläggande, sedan länge etablerad vetenskap

Uppslagsverk på webben

Wikipedia är ett flerspråkigt webbaserat uppslagsverk med i huvudsak fritt och öppet innehåll som utvecklas av sina användare (ofta benämnda wikipedianer).

Wikipedia har mer och mer blivit en standardkälla för information. De svenska versionen är ganska begränsad, och jag rekommenderar den bara för svenska förhållanden. Den engelska versionen är emellertid mycket omfattande. Wikipedia kritiseras ibland för att den skulle vara otillförlitlig eftersom vem som helst kan skriva artiklar. Det är ju precis detta som är Wikipedias styrka! Om något fel kommer in så rättas den snabbt av någon annan. Om det är oenighet i ett ämne så markeras detta ofta tydligt i artikeln. Wikipedia innehåller faktiskt inte mycket fler fel och saknad information än Nationalencyklopedin (NE) enligt en undersökning man gjort på Sveriges Radio. Wikipedia är naturligtvis inte den slutliga källan när det gäller komlicerade begrepp, men den är en utmärkt utgångspunkt.

Nationalencyklopedin (NE, Nationalencyklopedin ) har fördelen att artiklarna skrivits av experter på det aktuella området, så kvalitén är hög och jämn. Nackdelen är att nyheter kommer in mycket långsammare än i Wikipedia där "nördar" (positivt menat) bevakar allt som händer. Wikipedias styrka är initierade artiklar även i ganska udda ämnen. Den pedagogiska nivån är emellertid mycket varierande. Avvikelser från Wikipedias principer markeras emellertid oftast tydligt.

En annan fördel med Wikipedia jämfört med NE är att alla viktiga fakta skall ha en referens till en originalkälla. Om detta inte är fallet för artiklar med lägre kvalité så signaleras detta oftast längst upp på sidan.

En stor fördel med Wikipedia är att många bilder är helt fria under Wikimedia Commons .

Pseudovetenskap mm

Ett vanligt pseudovetenskapligt trick är att man påstår något som inte har något stöd i teori eller experiment, men som å andra sidan inte kan motbevisas. Detta kallas efter filosofen Bertrand Russell för Russells tekanna.

Russells tekanna eller den himmelska tekannan är en analogi av filosofen Bertrand Russell. Analogin är ett argument mot idén att det är en skeptikers uppgift att motbevisa religiösa dogmer, snarare än den troendes uppgift att bevisa dem, se Russells_tekanna .

Länk 1 innehåller artiklar om och exempel på pseudovetenskap. Nature of Science är en utmärkt interaktiv site om vetenskaplig metod. James Randi Educational Foundation är hemsidan för en av de mest kända förkämparna för vetenskaplig metod och mot pseudovetenskap. Wikipedia-artiklarna Vetenskap och Pseudovetenskap är mycket bra. Se även Russell's_teapot , Science , Scientific_method , Pathological_science och Pseudoscience .

Referenser: bland annat Carl Sagan: Demon-Hunted World, Robert L. Park: Voodoo Science: The Road from Foolishness to Fraud och Bennett, Shostak, Jakosky: Life in the Universe.

Se även nedanstående figur (Image credit: Hemant Mehta of the Friendly Atheist blog).



/Peter E

Nyckelord: pseudovetenskap [11]; vetenskaplig metod [18]; Wikipedia [5];

1 http://fragelada.fysik.org/links/search.asp?keyword=pseudovetenskap
2 http://arxiv.org/

*

Materiens innersta-Atomer-Kärnor, Universum-Solen-Planeterna [14333]

Fråga:
Sannolikheten att neutrinen skall träffa på en elektron eller atomkärna i sin resa genom materia är vad jag förstått så gott som obefintlig, det får mej att tro att det så kallade elektronmolnet måste vara tämligen glest, om man skall tolka uttrycket ”rakt igenom” atomen utan avvikelse. Sedan kan man väl anta att en kollision inträffar då och då utan för oss märkbart resultat. Finns det någon annan känd partikel som har samma förutsättningar att gå igenom materia (atomer) En annan fundering från frågelådans svar. Hur kunde man ”veta” att de 19 detekterade neutrinerna kom från supernova utbrottet, var det riktningen på spåren i detektorn, eller hur gick det till?
/Walter H, Rörö

Svar:
Nej, neutrinon är enligt standardmodellen den enda partikeln som bara växelverkar med den svaga växelverkan.

De 19 neutrinerna från supernovan 1987A kom dels nästan samtidigt och dels från rätt riktning, se nedanstående länkar.

Bilden nedan visar SN 1987A några år efter utbrottet, se Type_II_supernova och SN_1987A



/Peter E

Se även fråga 125

Nyckelord: supernova [13]; neutrino [19]; SN 1987A [4];

1 http://hyperphysics.phy-astr.gsu.edu/hbase/astro/sn87a.html#c2
2 http://www-personal.umich.edu/~jcv/imb/imbp4.html

*

Partiklar [14644]

Fråga:
hej! jag undrar: vad är en neutrino för något egentligen? hur påverkar dessa neutrinos oss och vår miljö, alltså vilka effekter ger denna elementerpartikel? Hur uppstår neutrinos? Varför har man forskat och studerat neutrinos noggrannt de senaste 20 åren? Jag vore väldigt tacksam om ni kunde svara på mina frågor i så enkel form som möjligt? Om ni kan tipsa mig om några bra svenska hemsidor eller böcker vore jag väldigt tacksam!
/isabelle j, segeltorpsskolan, segeltorp

Svar:
Isabelle (igen)! Börja med att läsa befintliga svar: neutrino och standardmodellen . De är av mycket varierande svårighetsgrad, men några kan du säkert förstå. Du bör bland annat se att trots att det finns så mycket neutroner, så är det inget vi behöver oroa oss för: de går igenom allting som spökpartiklar.
/Peter E

*

Materiens innersta-Atomer-Kärnor [14707]

Fråga:
Hej Lund! Jag har en fråga gällande elementarpartiklar. I en lärobok för fysik B i gymnasiet stod det att man tror att kvarkar är uppbyggda av något som man valt att kalla rishoner, en annan uppfattning är strängteorin. Jag antar att strängteorin är mer utbredd i dagens läge fast hur långt har man egentligen kommit på "rishonmodellen"?
/Oscar S, Östanå gymnasium, Eksjö

Svar:
Oscar! Jag har aldrig hört talas om rishoner, och jag tycker det är mycket konstigt om en så lite etablerad teori kommer in i en gymnasielärobok. Kvarkar är enligt standardmodellen elementarpariklar. Strängteori n är ännu inte i ett stadium där den kan förutsäga något.
/Peter E

*

Kraft-Rörelse [14921]

Fråga:
Beror friktionskraften av kontaktytan?
/Veckans fråga

Ursprunglig fråga:
Fick en förfrågan om friktionskraften är areaberoende. Känns som att det borde vara så att större area ger större friktion. Fast om jag kollar på formeln friktionskraften = friktionskoefficienten*normalkraften finns inget som visar på att friktionskraften skulle vara areaberoende. Är friktionskraften areaberoende? Hur förklarar man det? Hur förklarar man att vilofriktionen är större än glidfriktionen?
/Marianne A, Ehrensvärdska gymnasiet, Karlskrona

Svar:
Marianne! Friktionskraften är ett mycket komplicerat fenomen när man djupdyker i det, men standardmodellen är mycket enkel: friktionskraften f är proportionell mot normalkraften N:

f = mN

där proportionalitetskonstanten m kallas friktionskoefficient. Som du ser är det inget beroende av arean. Att det är så kan man intuitivt förstå eftersom om vi t.ex. dubblar arean så blir normalkraften per ytenhet hälften så stor, så resultatet blir oförändrat. Proportionaliteten mellan friktionskraften och normalkraften kan man förstå om man tänker på att den reella kontaktytan (utgörs av några atomer som sticker ut) är ganska liten. Om man ökar normalkraften så kommer atomerna att flytta sig lite, och fler atomer kommer i kontakt med varandra. Detta gör att friktionen ökar. Att den ökar linjärt kan enklast betraktas som ett experimentellt faktum.

Bilderna nedan (från Hyperphysics, länk 1) visar vilofriktion (statisk) och glidfriktion (kinetisk). Den förra är som du säger större. Anledningen är komplex och beror av materialet, men det har att göra med att knöligheter fastnar i varandra när klossen ligger still, medan denna effekt minskar om klossen rör sig.

I den nedre figuren visas friktionskraften f som funktion av den drivande kraften F. Klossen väger 10 kg, så normalkraften är hela tiden c:a 100 N. Till vänster (i origo) är friktionskoefficienten noll (ingen drivande kraft att motverka). Friktionskoefficienten ökar när man går åt höger tills den blir 0.5. Då övervinns friktionskraften och klossen börjar röra sig. Observera att så snart klossen sätts i rörelse så minskar friktionskoefficienten till (i det här exemplet) 0.4.

Se vidare länk 1 och friction för mer om friktion än du någonsin vill veta .



/Peter E

Nyckelord: friktion [53]; *vardagsfysik [64]; friktionskoefficient [5];

1 http://hyperphysics.phy-astr.gsu.edu/hbase/frict.html

*

Materiens innersta-Atomer-Kärnor [15152]

Fråga:
mesoner, är de budbärare eller vad, vilken kraft?

varför har svaga kraften så omständigt namn (inter mediär vektor boson) jämfört med de andra?
/alf e, acheberg, göteborg

Svar:
Mesoner används för att beskriva kraften mellan nukleoner i en kärna. Detta är inte en fundamental kraft (och mesonerna inte kraftförmedlingspartiklar enligt standardmodellen ) utan en yttring av den starka färgkraften som håller ihop kvarkar. Se fråga 14216 och länk 1 nedan.

Partiklarna kallas vektorboson för att spinnet är 1 (som alla kraftförmedlarpartiklar). Intermediär vet jag inte var det kommer ifrån. Partiklarna kallas numera helt enkelt W+, W- och Z0. Se länk 2.
/Peter E

Se även fråga 14216

Nyckelord: standardmodellen [24];

1 http://hands-on-cern.physto.se/hoc_v21sv/rollover/ro_pimesonkraft.html
2 http://hyperphysics.phy-astr.gsu.edu/hbase/particles/expar.html#c3

*

Materiens innersta-Atomer-Kärnor [15771]

Fråga:
Nu när den stora acceleratorn LHC startar på CERN: är det risk att det bildas ett svart hål som hela jorden kan försvinna i?
/Veckans fråga

Ursprunglig fråga:
Nu när den stora acceleratorn LHC startar på CERN: är det risk att det bildas ett svart hål som hela jorden kan försvinna i?
/Mamma till orolig lågstadieelev

Svar:
Nej, det är ingen fara! Det bildas inga svarta hål!

Förhoppningsvis kan man genom kollisioner åstadkomma materia med mycket hög temperatur och densitet, och denna skulle kunna ha andra egenskaper än vanlig materia - den skulle kunna tänkas innehålla fria kvarkar. Detta kallas kvark-gluon plasma. Men som sagt, det är helt ofarligt. Naturen utför ännu mer våldsamma experiment när kosmisk strålning träffar jorden. Det förekommer kärnpartiklar med energier upp till 1 joule i den kosmiska strålningen - detta är mycket mer än vad man kan åstadkomma med LHC.

Det andra man vill åstadkomma med LHC är att hitta den s.k. higgspartikeln . Denna behövs för den s.k. standardmodellen .

Länk 1 nedan är en artikel om LHC från Sydsvenskan. Mer information finns på LHC och CERN . Den senare har en artikel om säkerheten med LHC där det bland annat står:

“The LHC will enable us to study in detail what nature is doing all around us,” said CERN Director General Robert Aymar. “The LHC is safe, and any suggestion that it might present a risk is pure fiction.”

Tillägg 11 september 2008:
Det har på andra sätt kommit in en del frågor om farligheten hos LHC. Delvis baseras detta på information i en liten artikel av Johan Hakelius i Aftonbladet. Han är dock tydligt ironisk mot uppgifterna, och han kan ju inte hjälpa att några tossar förutspår världens undergång. Det har alltid funnits sådana, se t.ex. The_End_Is_Nigh#Origin_of_name .

Anton har en annan synpunkt förutom jordens undergång: skall man lägga ner miljarder euro för att bygga en accelerator som kanske kommer fram till något som knappast är relevant för de flesta människor? Ja, så länge man har en hygglig balans mellan direkt nyttig forskning och nyfikenhetsforskning så tycker jag det! Hade människan inte varit nyfiken så hade vi varit kvar i träddungarna i Afrika!

Många av de nyttigheter vi har idag (och för all del även onyttigheter) är direkta resultat av nyfikenhetsforskning. Ett exempel från CERN är World Wide Web. I dag är webben en vida spridd och viktig nyttighet: från början var det ett sätt för forskarna att kommunicera sina resultat tills Tim_Berners_Lee kom på att kanske hela värden behöver kommunicera!

Det viktigaste skälet för nyfikenhetsforskning är emellertid att människan är inte människa om hon inte får ägna sig åt annat än det direkt för överlevnaden nyttiga! Det är ju ändå så att en symfoni av Beethoven, en vacker tavla och en liten förståelse hur vi passar in i universum gör livet rikare!

Tack Madelene för Aftonbladet-artikeln och Nils-Göran för Antons inlägg!

Två intressanta videor:


/Peter E

Nyckelord: higgspartikeln [10]; standardmodellen [24]; gluoner [7];

1 http://sydsvenskan.se/varlden/article365220/De-storsta-svaren-finns-i-de-minsta-bitarna.html

*

Materiens innersta-Atomer-Kärnor [16082]

Fråga:
Hej! En partikels läge och rörelsemängd kan inte båda bestämmas med godtycklig noggrannhet enligt Heisenbergs osäkerhetsprincip. En foton har en per definition fastlagd hastighet och massa(?). Var "tar man ut osäkerheten"? Lägesbestämningen? Elektronneutrinerna är mycket lätta; de anses ha massa, men vad består den av , vet man det? Kan man säga något om neutriners rörelsemängd och fart??
/Thomas Å, Arlandagymnasiet, Märsta

Svar:
Det är ju partiklarnas vågegenskaper som ger upphov till Heisenbergs osäkerhetsprincip och sambandet mellan rörelsemängd och våglängd är detsamma (detta var ju de Broglies ansats, se fråga 13912 nedan).

Om neutrinerna har massa (vilket observerade oscillationer indikerar) kan dom inte röra sig med ljushastigheten. Å andra sidan är neutrinerna masslösa i standardmodellen. Se även fråga 125 nedan och Heisenbergs obestämdhetsrelation .
/Peter E

Se även fråga 13912 och fråga 125

*

Materiens innersta-Atomer-Kärnor [16236]

Fråga:
Hur har atomens nya uppbyggnad med kvarkar och strängar påverkat vårt sätt att se på vår omvärld? Tack i förhand!
/sofie b, göksten, eskilstuna

Svar:
Sofie! Inte ett dugg skulle jag säga! Strängteorin har ännu inga konsekvenser i observationer så den lever lite sitt eget liv.

Inte heller kvarkteorin (standardmodellen), som däremot är mycket väl etablerad, har några praktiska konsekvenser för kärnfysik och atomfysik. Man räknar fortfarande på atomkärnan som en samling neutroner/protoner. Först när man går till högre energier och studerar hadroner (protoner, neutroner, mesoner mm) kommer kvarkar och gluoner in i bilden.

Länk 1 är en kul sång, Bohemian Gravity (originalet Bohemian Rapsody av Queen), som bland annat handlar om strängteori.
/Peter E

Nyckelord: strängteori [7]; standardmodellen [24];

1 http://www.youtube.com/watch?v=2rjbtsX7twc&feature=youtu.be

*

Materiens innersta-Atomer-Kärnor [16404]

Fråga:
Hej! Några sönderfallsfrågor: Enligt en tabell är protonen inte säkert stabil, utana/men har en medellivslängd >_10^31 år. Universums nuvarande ålder är ju rätt mycket mindre än de tildslängden så någon större mängd sönderfall har väl knappast registrerats. Men vilka är de tänkta sönderfallsprodukterna? En partikel "delta" har medellivslängden ~10^-23 s, (finns begreppet naturlig kärntid än?) men saknar angivna sönderfallsprodukter. Är de okända eller blir de "bara energi" dvs strålning? Vart tar deras laddning i så fall vägen? Vilomassan för en fri kvark är okänd, men skulle en fri kvark vara stabil? Tauneutrinens stabilitet tycks ha varit ifrågasatt. Gäller detta än eller är den stabil numera?
/Thomas Å, Arlandagymnasiet, Märsta

Svar:
Thomas! Du vet vad jag tycker om flerdelade frågor !

Vid eventuellt brott mot baryontalets bevarande kan protonen sönderfalla så här:

p -> e+ + p0

se Proton_decay . Sönderfallet har ej observerats.

Delta-resonansen sönderfallet till en nukleon och en pion, se Delta_resonance . Laddningen måste bevaras.

Eftersom det inte finns fria kvarkar kan man inte säga något om stabiliteten. Antagligen skulle bara den lättaste kvarken vara stabil.

Enligt standardmodellen är tau-neutrinen stabil. Eftersom neutrinerna ganska säkert har massa, så kan man pga neutrino-oscillationer hävda att alla neutriner är instabila
/Peter E

*

Universum-Solen-Planeterna [17472]

Fråga:
Hej! Diametern på det synliga universum har angivits till 93 miljarder ljusår. Universums ålder anges till c:a 13,8 miljarer år. Om universum utvidgats med ljushastigheten borde storleken väl vara c:a 13 miljarder ljuår "i radie", dvs det dubbla i diameter. Diskrepansen är rätt stor. är orsakentill den den s k inflationen, som anses ha skett strax efter begynnelsen? Hur länge pågick inflationen, vad orsakade den (=hur kom den till) och vilken var dess hastighet -den tycks ju ha gått fortare än ljuset!-?
/Thomas Å, Knivsta

Svar:
Hej Thomas! Var har du fått 93 miljarder år från (se dock Observable_universe#Size )? Om universum är 14 miljarder år gammalt så är diametern hos det synliga universum definitionsvis 28 miljarder ljusår. Hur stort "hela" universum är vet vi helt enkelt inte, det kan vara oändligt stort. Se fråga 6116 .

Inflation är att universum under någon bråkdels sekund nästan direkt efter big bang expanderade extremt fort - storleken beräknas ha ökat ungefär 1028 gånger, se Inflation_(cosmology) .

Expansionen vid inflationen var mycket kortvarig omkring 10-38 s.

Inflationsperioden var mycket kortvarig från 10−36 sekunder efter Big Bang till mellan 10−33 och 10−32 sekunder efter Big Bang.

Inflationen skedde med en hastighet som översteg ljushastigheten. Orsaken kan vara en fasförändring (som frigjorde energi) då den starka (färg)kraften skildes från den elektrosvaga, se fråga 1496 . Se länk 1. De flesta kosmologer anser att inflationen som sådan är väl etablerad. Tidpunkterna är emellertid osäkra - vi har inga observationer från så nära Big Bang. Det finns förhoppningar att detta kan ändras med utvecklandet av mer känsliga gravitationsvågsdetektorer.

Bilden nedan av de fyra olika kraftverkningarna är från länk 1. I standardmodellen finns fyra kraftverkningar:

* gravitation
* svag växelverkan (betasönderfall)
* elektromagnetism
* starka färgkraften (QCD - håller ihop kvarkar)

Dessa kraftverkningar var vid big bang förenade i en kraft. Efter hand har ur-kraften separerats till fyra olika krafter. Kärnkraften (som håller ihop protoner och neutroner i atomkärnan) är inte en separat kraft utan en yttring av den starka färgkraften.



/Peter E

Nyckelord: inflation [7]; big bang [37]; standardmodellen [24]; kraftverkningar [9]; gravitationsvågor [19];

1 http://www.daviddarling.info/encyclopedia/B/Big_Bang.html
2 http://aether.lbl.gov/www/science/inflation-beginners.html

*

Partiklar [17502]

Fråga:
Antimateria på CERN
/Veckans fråga

Ursprunglig fråga:
Hej! I CERN har det producerats antimateria, enligt tidningsrapporter. Har antimateriepartiklarna samma kvanttal som vanlig materia? Följer antimaterian Paulipricipen? Hur skiljer/påvisar man en neutron från en antineutron, de är ju båda oladdade?
/Thomas Å, Knivsta

Svar:
Thomas! Det nya är att man lyckats "klä på" antiprotoner med positroner och alltså lyckats framställa ett litet antal (38) anti-väteatomer. Det innebär att man kan studera övergångar i anti-väte och jämföra dem med väte. Standardmodellen säger att egenskaperna skall vara exakt desamma med undantag för laddningen. Antipartiklar följer Pauliprincipen mot andra identiska antipartiklar, mot motsvarande partiklar saknar Pauliprincipen mening.

Man alltså lyckats framställa en liten mängd oladdad antimateria. Än så länge inte tillräckligt för att scenariot i Dan Browns bok (och filmen) Änglar och demoner skall bli verklighet!

Neutronen har t.ex. ett magnetiskt moment. Anti-neutronen har det motsatta eftersom kvarkarna har annan laddning. Neutronen har kvarksammansättningen ddu (laddning -1/3,-1/3,+2/3). Antineutronen har sammansättningen dantidantiuanti (laddning +1/3,+1/3,-2/3).

Se vidare "over the top" artikeln länk 1 och den mer sansade pressreleasen länk 2.
/Peter E

Nyckelord: standardmodellen [24]; antimateria [16]; kvark [12];

1 http://www.theregister.co.uk/2010/11/18/cern_antimatter_bomb/
2 http://press.web.cern.ch/press/PressReleases/Releases2010/PR22.10E.html

*

Partiklar [17518]

Fråga:
Neutrinooscillationer
/Veckans fråga

Ursprunglig fråga:
Hej! neutrinooscillation innebär att de olika neutrintyperna kan övergå i varandra, om jag läst rätt. Men de tycks ha olika vilomassor. Var kommer massan från om en lätt neutrin övergår till/i en tyngre? Från hastigheten(kinetisk energi blir massa)? Och om en tung omvandlas till en lätt, ökar den farten då eller vart tar massan vägen? Hur/När omvandlingen sker, är det bekant? /Hur bör jag förklara det för elever?!/
/Thomas Å, Knivsta

Svar:
Nobelpris i fysik 2015
Neutrinooscillationer
Takaaki Kajita, Arthur B. McDonald
”för upptäckten av neutrinooscillationer, som visar att neutriner har massa”

Neutrino: Neutrinon är en elementarpartikel, som tillhör gruppen leptoner och saknar elektrisk laddning. Den har halvtaligt spinn och är därför en fermion. Neutrinon har mycket liten massa och är universums mest förekommande partikel.

Neutrinooscillationer är ett fenomen i elementarpartikelfysiken som innebär att neutriner, som kan skapas och detekteras i tre väl definierade skilda slag (aromer) kan ändra karaktär på väg från källa till detektor.

Neutrinooscillationer kan inträffa om elektron-, myon- och tauneutriner har olika massa, vilket innebär att de inte alla kan vara masslösa. I partikelfysikens standardmodell är neutrinerna exakt masslösa. Vittnesbörd om neutrinooscillationer är därför ett tecken på ny fysik bortom Standardmodellen. (Från svenska Wikipedia)

Neutrinoocillation är ett kvantmekaniskt fenomen, och som vanligt när det gäller dessa är det svårt (på en fundamental nivå omöjligt) att förstå, se Neutrino_oscillations . Lite kan man dock förstå relativt enkelt:

De tre aromerna av neutriner är sammansatta av tre neutriner med olika massa (m1, m2 och m3):

|elektron> = e1|m1> + e2|m2> + e3|m3>
|myon> = m1|m1> + m2|m2> + m3|m3>
|tauon> = t1|m1> + t2|m2> + t3|m3>

Massdifferenserna mellan typ 1, 2 och 3 är mycket små - bråkdelar av eV. Energier hos typiska neutriner vi observerar är MeV eller GeV. Men det är den totala energin vi måste bevara - det finns inget som säger att vilomassan skall bevaras. Vi behöver alltså "fuska" med energier av storleksordningen meV när vi har MeV tillgängligt. För att uppfylla bevarandet av energin rör sig de olika egentillstånden (som ju har lite olika massor) med lite olika hastighet.

Det finns en klassisk mekanisk analog till neutrinooscillation: två kopplade pendlar, se Neutrino_oscillations#Classical_analogue_of_neutrino_oscillation .

Det finns andra exempel från kvantmekaniken där två tätt liggande nivåer blandas och förorsakar interferens och att systemet oscillerar mellan två mycket olika tillstånd.

I fråga 125 beskrivs en annan observation som tyder på att neutrinon har en vilomassa som är större än noll.
/Peter E

Nyckelord: kvantmekanik [30]; neutrino [19];

1 http://fof.se/tidning/2015/11/artikel/neutriner#overlay=tidning/2015/11/artikel/neutriner
2 http://www.svt.se/nyheter/vetenskap/nobelpriset-i-fysik-2015

*

Partiklar [17620]

Fråga:
Hej! Kallas kvarkar för elementarpartiklar? Gör neutroner och protoner det fortfarande fast de består av mindre partiklar i form av kvarkar?
/Sarah A, Lillerudsgymnasiet, Vålberg

Svar:
Sarah! Ja, kvarkar är elementarpartiklar enligt standardmodellen . Nej, protoner och neutroner kallas inte elementarpartiklar i dag eftersom de är sammansatta av kvarkar.

Figuren i fråga 9598 visar alla elementarpartiklar enligt standardmodellen.
/Peter E

*

Materiens innersta-Atomer-Kärnor [17763]

Fråga:
Hej! I en länk till en länk i ett frågesvar fanns en tabell där gluoner skrevs ha vilomassan = 0. Det finns ju begreppet kvark-gluonplasma, där hög fart väl är rådande (det är ju hett) men eljest förknippar jag gluonerna med atomkärnan och den är rätt liten. Innebär masslösheten då att de alltid skulle röra sig med ljushastigheten? Även inuti atomkärnan?

Vilomasslösa partiklar syns vara fotonen, gravitonen och gluonen. De skall då röra sig med ljushastigheten, c. Fotonens energi/egenskaper bestäms av dess frekvens. Vilka egenskaper bestämmer ev energi/egenskaper hos de andra två förmedlarpartiklarna?
/Thomas Å, Knivsta

Svar:
Thomas! En länk till en länk! "Your honour I protest, this is hearsay!" Det finns emellertid en förstahandskälla: Gluon . Vilomassan för gluonen är 0 enligt standardmodellen.

Ja, om gluonerna är masslösa har de alltid hastigheten c. Normalt är kvarkarna och gluonerna (som håller ihop kvarkarna med vad som kallas den starka färgkraften, se nedanstående länkar) inneslutna i utåt färglösa system. Vid mycket hög energi (temperatur) kan man tänka sig att man får en soppa med fria kvarkar och gluoner. Det är detta som kallas kvark-gluon plasma, se Quark–gluon_plasma .

Van gäller vilomasslösa partiklar, se fråga 13912 . Det är alltså våglängden hos materievågorna som ger den totala energin.
/Peter E

Nyckelord: standardmodellen [24]; färgkraften [8]; gluoner [7];

*

Universum-Solen-Planeterna [18733]

Fråga:
Hej! Vilka är de andra, icke omöjliga modeller av universum som avses i svaret på innevarande "Veckans fråga"? Var får man veta mer om dem?
/Thomas Å, Knivsta

Svar:
Du refererar till fråga 18686 .

Svaret var inte bra formulerat när det gäller andra teorier. Det finns varianter av standardmodellen, men knappast någon accepterad som alternativ. Sedan vill ju sträng-teoretikerna få ihop en teori, men den finns inte i dag. Det viktigaste jag ville ha fram var att parametervärdena är modellberoende.
/Peter E

*

Partiklar [18752]

Fråga:
Hej, jag har några frågor om higgs partikel. Vad har partikeln för anknytning till Big Bang? Och varför kallas den gudspartikel?
/Anna H

Svar:
Higgspartikeln bildades strax efter big bang när den elektrosvaga kraften delades upp i den svaga och den elektromagnetiska kraften, se länk 1.

Den kallas gudspartikeln eftersom den är så viktig genom att skapa massa hos de andra partiklarna i standardmodellen, se Higgsboson och på engelska Higgs_boson och The_God_Particle:_If_the_Universe_Is_the_Answer,_What_Is_the_Question? .

Se även dokument under länk 2.
/Peter E

Nyckelord: higgspartikeln [10]; standardmodellen [24];

1 http://public.web.cern.ch/public/en/Science/Higgs-en.html
2 http://www.lth.se/fysik/om_institutionen/nyheter/nyheter_juli_2012/higgspartikeln_artiklar_i_urval/

*

Blandat [18849]

Fråga:
Hur har upptäckten av standardmodellen påverkat människan?
/Veckans fråga

Ursprunglig fråga:
Hejsan! Hur har upptäckten av standardmodellen påverkat människan? Vad är de positiva resp. negativa effekterna av upptäckten?
/Agnes H, Donnergymnasiet, Göteborg

Svar:
Agnes! Med standardmodellen inom partikelfysik avses den modell som beskriver de minsta partiklarna och deras interaktioner (kraftverkningar) genom elektromagnetisk, stark och svag växelverkan med hjälp av kvantfältteori. Standardmodellen är en kvantmekanisk teori, men den är ingen heltäckande modell eftersom den inte innefattar gravitationskraften. Se bilden nedan och Standardmodellen .

Standardmodellen kom inte till genom att en person hade en bra idé. Den kom till genom att man under 50-60-talen med hjälp av allt större acceleratorer upptäckte fler och fler partiklar. Till sist blev det en förfärlig röra med partiklar. Då upptäckte några fysiker ett mönster i partiklarnas egenskaper genom att klassificera partiklarna med avseende på olika egenskaper. Man kunde då se att vissa platser i schemat var tomma. Då fick man en antydan till vad man skulle leta efter. Nu har man hittat alla partiklar i det ursprungliga schemat. Om det finns fler partiklar än de som omfattas av standardmodellen vet man inte.

I standardmodellen ingår två typer av partiklar, leptoner och kvarkar. Dessutom ingår higgspartikeln (som har att göra med elementarpartiklarnas massor) och kraftförmedlingspartiklar.

Lepton: Leptonerna är den ena huvudgruppen av fermioniska (med halvtaligt spinn) elementarpartiklar. Av leptoner finns det sex stycken aromer, vilka liksom kvarkarna delas in i tre familjer. Varje familj består av en partikel och tillhörande neutrino. Se Lepton .

En kvark är en elementarpartikel som tillsammans med en eller flera andra kvarkar bygger upp den grupp partiklar som kallas hadroner (t.ex. nukleoner och mesoner). Så vitt man vet idag är kvarkarna, tillsammans med leptonerna, materiens minsta byggstenar. Det finns sex olika typer av kvarkar, kända som aromer. Aromerna med den lägsta massan, uppkvarken och nedkvarken, är i allmänhet stabila och mycket vanligt förekommande i universum. Se Quark .

Den viktiga skillnaden mellan leptoner och kvarkar är att leptoner växelverkar bara med den svaga kraften medan kvarkar växelverkar med både den svaga och starka kraften.

Här är en kul föreläsning om hur atomer och kvarkar är uppbyggda:

Eftersom mycket få personer fullt ut förstår standardmodellen är dess praktiska betydelse mycket begränsad. Det ligger emellertid i människans natur att försöka förstå världen omkring sig, och för detta är standardmodellen tillsammans med big bang teorin mycket viktiga puzzelbitar. Även om man inte förstår teorierna fullständigt, kan det vär vara intressant att veta att universum är uppbyggt av kvarkar och leptoner och att universum skapades i en gigantisk explosion för 13.7 miljarder år sedan...?

Ett annat skäl att bedriva avancerad forskning t.ex. på CERN är att man utvecklar ny teknik som kan användas för andra ändamål. Ett exempel är avbildning för medicinsk diagnos (PET, se Positron_emission_tomography ).

Ett annan utveckling är när Tim Berners-Lee (Tim_Berners-Lee ) på CERN ville åstadkomma ett informationssystem som forskarna kunde använda för snabb kommunikation av data och idéer. Det dröjde inte länge innan man insåg att detta system hade generell tillämpbarhet, och WWW (World_Wide_Web ) var fött.

Avancerad forskning är även utmärkt för att utbilda nästa generations forskare, lärare, industriledare och entreprenörer.

Några negativa effekter av grundläggande forskning är svårare att hitta. Möjligen om man använder kunskapen till destruktiva tillämpningar, t.ex. vapen. Men då är det tillämpningarna som är destruktiva, inte kunskapen. Sedan kan man även hävda att frontilinjeforskning inom vissa områden (t.ex. partikelfysik, astrofysik) är dyr eftersom den kräver stora och avancerade apparater.

/*fa*



/Peter E

Nyckelord: fysik, nytta med [6]; standardmodellen [24];

*

Partiklar [18943]

Fråga:
Hej! Kan ni vara snälla att svara på några frågor så att en 13 åring kan förstå?

1 Hur gör man antimateria?

2 Hur kan man jobba med antimateria utan att allt exploderar?

3 Varför exploderar antimateria och vanlig materia om de kommer i kontakt med varandra?

4 Kan det bildas planeter stenar och solar av antimateria?

5 Det verkar finnas en massa olika teorier om varför det finns mer materia än antimateria.

Vilken av dem är troligast?

6 Finns det andra sorters materia?
/Ronja B, färe, osby

Svar:
Ronja! Jag skall försöka.

1 Man accelererar partiklar med hjälp av en accelerator, se accelerator och Particle_accelerator . Ofta använder man kolliderare där man låter pariklarna gå runt åt båda hållen och kollidera i några punkter. Vid kollisionerna bildas partikel/antipartikelpar i en process som kallas parbildning .

2 Om bara antipartiklarna är stabila (t.ex. antiprotoner) räcker det att hålla dem separerade från materia. För detta lagrar man antipartiklarna i en Lagringsring som är ett lufttomt rör med magneter. För antiprotoner hade man på CERN en ring som kallades LEAR Low_Energy_Antiproton_Ring .

3 Om en partikel och en antipartikel kommer nära varandra så förintar de varandra i en process som kallas annihilation . Energin blir till gammastrålning eller partikel/antipartikelpar.

4 I princip ja, om antimaterian är separerad från materian. Men det är ganska säkert att det inte finns stora mängder antimateria i universum.

5 Ja, det finns flera teorier, men ingen allmänt accepterad och de är alla svåra att förstå. Man kan säga att om det inte varit ett litet överskott av materia så hade vi inte funnits för att kunna fundera på detta. Universum hade bestått av bara strålning. De teorier som finns är inte lätta att förstå: Baryon_asymmetry .

6 Nej, inte enligt standardmodellen (se fråga 18849 ) som är allmänt accepterad.

Se vidare 16426 , 17502 och nedanstående länkar.
/Peter E

Nyckelord: antimateria [16]; annihilation [14]; parbildning [7];

*

Partiklar [19037]

Fråga:
Hur ska jag kunna förmedla på bästa sätt för min klass om partikelfysik? Har du några intressanta områden man kan ta upp i den stora partikelfysikens värld?
/Marcus E, SALA

Svar:
Det finns massor med intressanta ämnen, till exempel:

standardmodellen
färgkraften
higgspartikeln
gluoner
kvark
kärnkrafter
neutrino
/Peter E

*

Partiklar [19140]

Fråga:
Hej! I en tidningsartikel om Cern står det att standardmodellen med upptäckten av Higgspartikeln nu är komplett, men att man ändå letar vidare efter nya partiklar, som skulle tillhöra "Higgsfamiljen", fem eller sju nya partiklar. (Eller hitta en helt ny fysik.) Jag har aldrig hört tals om att det skulle finnas fler än en sorts higgspartikel(än den som ger materia massa). Vad skall/skulle de oupptäckta ha för funktion eller ge svar på för frågor?
/Thomas Å, Knivsta

Svar:
Nej, standardmodellen är inte komplett. I bästa fall är den ofullständig, i sämsta helt enkelt fel. Bland annat saknas gravitationen, och neutrinerna i modellen har ingen massa medan de enligt övertygande observationer (neutrino-oscillationer) har en liten massa. Det finns också varianter av Higgs teori med 1-7 higgspartiklar.

Vilka frågor detta skulle besvara är svårt att se: det bara är så. Varför finns det t.ex. två extra familjer elementarpartiklar (sär/charm och bottom/top)? En sak man letar efter är vad är den mörka materian som krävs för att den kosmologiska standardmodellen skall gå ihop.

Se vidare länk 1. Länk 2 är 2013 års Nobelföreläsning om higgspartikeln.
/Peter E

Nyckelord: standardmodellen [24]; higgspartikeln [10];

1 http://www.svd.se/nyheter/utrikes/cern-jublar-i-skuggan-av-ny-partikel_7320327.svd
2 http://www.nobelprize.org/nobel_prizes/physics/laureates/2013/englert-lecture.html

*

Partiklar [19172]

Fråga:
Vilka partiklar är stabila?
/Veckans fråga

Ursprunglig fråga:
Hej! Vilka partiklar är stabila? Elektronen är det och protonen. Men anses kvarkar stabila? En nerkvark övergår i uppkvark när en neutron sönderfaller, så nerkvarken är väl inte stabil? Och är neutriner stabila, eller anses de sönderfalla? (De är ju rätt energirika, så de borde kunna sönderfalla.)
/Thomas Å, Knivsta

Svar:
Nedanstående bild visar alla elementarpartiklar enligt standardmodellen. Partiklarna är vertikalt ordnade efter vilomassa.

För att en partikel skall sönderfalla måste ett antal villkor (baserade på bevaringslagar) uppfyllas:

1 Sluttillståndet måste innehålla minst två partiklar
2 Partiklarna i sluttillståndet måste ha en sammanlagd vilomassa som är mindre än den sönderfallande partikelns massa (bevarande av energin)
3 Totala laddningen måste bevaras
4 Antalet fermioner (partiklar med halvtaligt spinn) kan bara ändras med ett jämnt antal (bevarande av rörelsemängdsmoment)
5 Antalet kvarkar bevaras (bevarande av baryontal)

De partiklar som uppfyller dessa villkor är

1 masslösa kraftförmedlarpartiklar: foton och gluon
2 elektron
3 proton
4 neutrino

Listan kräver några kommentarer:

Protonen är ingen elementarpartikel eftersom den består av tre kvarkar. Kvarkar kan emellertid inte förekomma isolerade, varför protonen kan betraktas som elementarpartikel. Protonens stabilitet implicerar då att den lättaste kvarkarna (upp och ner) är stabila i protonkonfigurationen (upp, upp, ner). Neutronen (upp, ner, ner) är emellertid inte stabil om den är isolerad. Den b-sönderfaller till en proton, se fråga 17998 . Tillsammans med rätt antal protoner kan neutronen emellertid vara stabil i en atomkärna.

Lägg även märke till att de två lättaste kvarkarna har mycket mindre massa än protoner och neutroner med massan c:a 1 GeV. Nukleonerna består alltså av mycket mer än tre kvarkar, den största delen av massan kommer från gluoner (Quark#Mass ).

Neutrinon är alltså i princip stabil, men den oscillerar mellan olika aromer (elektron, myon, tau), se Neutrino_oscillation .

Se även Particle_decay

Bilden nedan är från http://profmattstrassler.files.wordpress.com/2011/08/sm_masses2.png



/Peter E

Nyckelord: standardmodellen [24]; kvark [12]; neutrino [19];

1 http://profmattstrassler.com/articles-and-posts/particle-physics-basics/why-do-particles-decay/most-particles-decay-why/
2 http://profmattstrassler.com/articles-and-posts/particle-physics-basics/why-do-particles-decay/most-particles-decay-yet-some-dont/

*

Universum-Solen-Planeterna [19289]

Fråga:
Hej!

Jag går just nu tredje året på gymnasiet och för att erhålla ett examensintyg krävs att man utför ett gymnasiearbete. Som gymnasiearbete valde jag att studera olika teorier som presenterats om universums undergång (The big freeze, big rip samt the big crunch). Min lärare föreslog att jag också skulle fråga en expert inom området och därför frågar jag här. Vilken av dessa teorier anser ni/du är mest rimlig just nu med tanke på nya upptäckter som gjorts sedan dessa teorier presenterades samt varför?

MvH Daniel
/Daniel G, Procivitas, Växjö

Svar:
Daniel!

Någon kosmologiexpert har vi inte här, men lite kan jag säga.

Big freeze
Expansionen fortsätter för alltid i en accelererande takt (på grund av den mörka energin). Universum får lägre densitet och blir kallare. Därav namnet. Se Future_of_an_expanding_universe .

Big Crunch
Universum har tillräcklig densitet för att gravitationen skall kunna stoppa expansionen. Universum kontraheras då i vad som kallats Stora krossen till ett mycket koncentrerat tillstånd. Eventuellt "studsar" universum tillbaka genom en ny Big bang. Se Big_Crunch .

Big Rip
Universum expanderar så snabbt att det observerbara universum blir mindre än galaxer, planetsystem och till sist atomer. I avsaknad av attraherande krafter kommer alltså allt att slitas sönder. Se Big_Rip .

Big Crunch stämmer inte med observationen av accelerationen hos expansionen. Big Rip anses nog ganska spekulativ. Big freeze är den kosmologiska standardmodellen, så det är den de flesta tror på.
/Peter E

Nyckelord: big bang [37];

*

Partiklar [19329]

Fråga:
Hejsan! Jag är en elev som pluggar sista året i naturvetenskap och jag har några funderingar över Wips partikel.

Jag fattar inte meningen med partikeln. Vad gör den? Hur har de kommit fram att det finns detta partikel? vilka konsekvenser har i Modern Fysik? vad har det här med förståelse av Universum att göra?

Jag är tacksam för svar

Hälsningar Aran
/Aran S

Svar:
Hej Aran! Jag antar du menar WIMPs.

WIMPs är hypotetiska partiklar som skall förklara förekomsten av mörk materia, se fråga 12396 . Se även WIMPs .

Man har letat men ännu inte funnit några WIMPs, men mörk materia är en mycket viktig del i den kosmologiska standardmodellen (Big Bang), se fråga 18686 .
/Peter E

*

Partiklar [19621]

Fråga:
Hur har man har kommit fram till Standardmodellen? Hur har fysiker har hittat de olika partiklarna som finns i Standardmodellen?
/Julia S, Kitas, Göteborg

Svar:
Julia! Vi har redan massor med svar om standardmodellen, se standardmodellen . De flesta av dina undringar bör besvaras där.
/Peter E

*

Universum-Solen-Planeterna [19655]

Fråga:
Hej

Jag har en teori om universums uppkomst/livet inan big bang. Min teori bygger på svarta håll. Tanken är att ett svart hål äter upp all massa (+ andra svarta hål) i universum och när svarta hållet inte kan äta mer så exploderar den (eller typ får "förhög" massa så att de går sönder), och då blir det ett nyt big bang och så fortsätter det så???

Men ett motargumen för detta är ju om svarta hål kan dö. Men det är därför jag frågar, för att få svar på om detta kan vara en rimlig/lågisk förklaring.
/Hjalmar K, Tomtaklintskolan, Trosa

Svar:
Hej Hjalmar! Det är bra att du funderar i nya banor. I standardmodellen anser man att tiden börjar vid Big Bang och att man inte kan säga något om vad som eventuellt hänt före Big Bang.

Stora svarta hål "dör" knappast, det är de små som kan försvinna, se fråga 19164 om hawkingstrålning.

Standardmodellen behandlas i fråga 18686 . I fråga 18978 beskrivs hur universum kan uppstå från i stort sett ingenting. Fler kosmologifrågor finns här: kosmologi .
/Peter E

*

Materiens innersta-Atomer-Kärnor [19656]

Fråga:
Hej! Vi håller på med kärnfysik på NOn just nu och jag hade en fråga angående betapartiklar som bildas i atomen då radioaktivt sönderfall sker. I min bok står det kort att "Man får tänka sig att en av neutronerna förvandlas till en proton och en elektron". Men jag har lite svårt att acceptera det svaret. Jag undrar vad det är som händer under själva processen när elektronen/protonen uppstår och hur en neutral partikel kan ge upphov till två laddade? Jag söker helt enkelt en lite mer ingående förklaring än den i vår fysikbok... Tacksam för svar!
/Mirjam S, Vallhallaskolan, Oskarshamn

Svar:
Så länge den totala laddningen bevaras är det inget problem med att en neutral partikel sönderfaller till laddade. Men om man tittar djupare (se nedan) är det faktiskt laddade partiklar (u- och d-kvarkar) involverade.

Se fråga 13471 för allmänt om betasönderfall. I fråga 12985 beskrivs processen i termer av den moderna bilden (standardmodellen, se fråga 18849 ) av den svaga växelverkan.
/Peter E

*

Partiklar [20015]

Fråga:
Higgspartikeln
/Veckans fråga

Ursprunglig fråga:
Hej! Elektriska fält, magnetiska fält och gravitationsfält är bekanta och påvisbara. Det s k Higgsfältet har tillkommit. Hur påvisar man det? Finns områden där det saknas eller är starkare?
/Thomas Å, Knivsta

Svar:
Higgsbosonen (även: Higgs boson eller Higgspartikeln) är en partikel i partikelfysikens standardmodell, som genom Higgsmekanismen och Higgs-fältet beskriver varför partiklar har massa. Vid ett seminarium vid CERN i början av juli 2012 tillkännagav talespersonen Fabiola Gianotti att man sannolikt upptäckt Higgsbosonen. (Higgsboson )

Att elementarpartiklar får massa genom växelverkan med Higgsfältet kan liknas vid att fältet utsätter partiklarna för friktion. Higgsfältet genomsyrar vakuum. Ett vakuum som rubbas ger upphov till en vågrörelse hos Higgsfältet och "vågtopparna" är det som kallas för Higgspartikeln eller Higgsbosonen. Den skalära Higgsbosonen förmedlar växelverkan mellan Higgsfältet och elementarpartiklar, som utgörs av kvarkar och leptoner och bosoner. Att olika partiklar har olika massa har populärt beskrivits som djup snö, där vissa partiklar vadar i motstånd, medan andra kan glida ovanpå. (Wikipedia Higgsmekanismen )

Higgsfältet är inte som andra fält i kvantfysiken. De andra, som gravitation, varierar i styrka, och när de hamnar i sin lägsta energinivå antar de värdet noll. Så fungerar inte Higgsfältet. Till och med om rymden töms på allt och bara vakuum blir kvar, så är detta vakuum fortfarande fyllt med en substans, ett spöklikt fält som vägrar att stängas av – Higgsfältet. Vi märker det inte – Higgs­fältet är som luft för oss, som vatten för fiskarna. Men utan det skulle vi inte finnas, för det är genom att doppas i Higgsfältet som partiklarna får sin massa. De partiklar som far igenom utan att märka Higgsfältet får ingen massa, de som växelverkar svagt blir lätta och några blir, när de segar sig igenom fältet, riktigt tunga. (länk 1)

Frågan är, Thomas, hur mycket klokare man blir av det . Ett problem är att massa uppenbarligen kopplas till gravitation medan gravitationen inte finns med i standardmodellen.

Här är en föreläsning av Leonard Susskind, Stanford University om standardmodellen och hur higgspartikeln ger elementarpartiklarna massa.

Länk 2 är nobelkommitténs som vanligt utmärkta populära beskrivning av standardmodellen och higgspartikelns betydelse.
/Peter E

Nyckelord: standardmodellen [24]; higgspartikeln [10]; nyheter [11];

1 http://fof.se/tidning/2013/6/artikel/i-morkret-bortom-higgs
2 https://www.nobelprize.org/nobel_prizes/physics/laureates/2013/popular.html

*

Universum-Solen-Planeterna [20034]

Fråga:
I den sk fluktuationsspektrumkurvan (ex i Max Tegmarks "Vårt Matematiska Universum" och Ulf Danielssons "Mörkret vid Tidens Ände") är Y-axeln temp-fluktuaton och X-axeln Multipol Moment. Vad är Multipol? och vad säger oss Kurvan?
/Björn G, Sundborn

Svar:
Björn! En detaljerad förståelse kräver en del, men en kvalitativ förståelse är inte oöverstiglig.

Temperaturen hos den kosmiska bakgrundsstrålningen är lite olika i olika riktningar. Temperaturen är alltså en funktion av två vinklar q,f. (Man använder normalt galaktiska koordinater.) För att specifiera uppmätta värden skulle det erfordras att man definierar temperaturen i oändligt många punkter. Detta är naturligtvis opraktiskt.

I stället representerar man temperaturen med en summa av vinkelfunktioner. Detta är analogt med att t.ex. representera trigonometriska funktioner med en Taylor-utveckling, se Taylorserie .

För funktioner som beror av vinklar använder man normalt klotytefunktioner, se Klotytefunktion för utvecklingen. Klotytefunktionen har två parametrar, grad (multipol eller multipolmoment) l och ordning m.

Figuren du refererar till visas nedan (länk 1 från ESA). Eftersom tolkningen i termer av multipoler (övre x-axeln) är lite svår att förstå har man även lagt in en skala (nedre x-axeln) som representerar den typiska "bubbelstorleken".

De röda punkterna i figuren är mätningar från Planck-proben. Den gröna kurvan är den bästa anpassningen av parametrarna i den kosmologiska standardmodellen. Man måste säga att överensstämmelsen mellan teori och uppmätta data är imponerande!

Vi ser att det finns en stor topp vid c:a 1o. Detta kan man tolka som att vårt universum är plant -- se resonemanget i fråga 18978 .

Se vidare länk 1 och 2.



/Peter E

Nyckelord: big bang [37]; kosmisk bakgrundsstrålning [19]; kosmologi [33];

1 http://sci.esa.int/planck/51555-planck-power-spectrum-of-temperature-fluctuations-in-the-cosmic-microwave-background/
2 http://sci.esa.int/planck/51562-the-power-spectrum-of-temperature-fluctuations-in-the-cosmic-microwave-background-ndash-animation/

*

Universum-Solen-Planeterna [20772]

Fråga:
Var precis inne och kollade på frågelådan. Där kunde jag läsa att ca 4% av all materia var känd materia resten någon form av mörk osynlig materia. På andra ställen har jag sett och på någon film hört ca 20% är känd materia och resten osynlig, mörk materia. Hur förhåller det sig egentligen?
/Annika V, Björnekullaskolan, Åstorp

Svar:
Det kan tyckas motsägelsefullt, men det är korrekt. Skillnaden är om man även räknar den mörka energin som massa eller hanterar den separat.

En anpassning av parametrarna i den kosmologiska standardmodellen (big bang) till vitt skilda observationer (kosmiska bakgrundsstrålningen, elementförekomst strax efter big bang, accelererad expansion från mörk energi, mm), se fråga 18686 , ger följande resultat

andel mörk energi 72.8%
andel mörk materia 22.7%
andel normal materia 4.56%

Bortser vi från den mörka energin är andelen normal materia
4.56/(4.56+22.7)=17%.
/Peter E

Nyckelord: big bang [37]; kosmologi [33]; mörk energi [6]; mörk materia [17]; materia [6];

*

Partiklar [21001]

Fråga:
Hej,

Undrar hur fria elektroner färdas då de inte interagerar med omgivingen, kontinuerligt eller diskontinuerligt? Mera generellt om elementarpartiklar med massa någonsin accelererar utan att de kanske bara efter interaktion med andra partiklar eller fält stegvis har högre eller lägre hastighet? Mera vilt undrande ifall standardmodellen kan beskriva gravitation utan allmänna relativitetsteorin då den till skillnad mot den speciella beskriver rörelser med acceleration ...

Med vänlig hälsning Mikael Hjelm
/Mikael H, Västerås

Svar:
Man kan inte tala om att en elektron rör sig i en viss bana. Elektronen har ju även enligt kvantmekaniken vågegenskaper. Dubbelspaltexperimentet, se fråga 1807 , är ett bra exempel på detta.

Nej, standarmodellen beskriver inte gravitation. Vi behöver den allmänna relativitetsteorin än så länge. Se fråga 18849 .
/Peter E

*

Materiens innersta-Atomer-Kärnor [21231]

Fråga:
Vad är laddning hos en elementarpartikel? Vilka oladdade finns det?
/Veckans fråga

Ursprunglig fråga:
Hej! Jag undrar lite om elementarpartiklar. En elementarpartikel har alltid en laddning, men vad är laddningen egentligen? Är det att energin/massan är positiv/negativ eller är det bara något "magiskt" som händer med partikeln? Samt hur kan jag se på en partikel vilken laddning den har?
/Elin E, Lund

Svar:
Elementarpartiklar är materiens minsta beståndsdelar. Partiklar som har mindre beståndsdelar räknas inte som elementarpartiklar. Till elementarpartiklarna räknar man också de partiklar som är bärare av de fyra fundamentala krafterna i naturen. Elementarpartiklar studeras inom partikelfysiken, där de partiklar man för närvarande känner till beskrivs av den så kallade Standardmodellen. (Elementarpartikel , Standardmodellen )

Det finns ett antal elementarpartiklar som är oladdade, nämligen tre neutriner, fotonen och Z-bozonen. Vad gäller övriga partiklar så har kvarkar +-(1/3)e laddningar och laddade leptoner laddningen +-e.

Laddade partiklar påverkar varandra med den elektromagnetiska kraften. I artikeln Elektrisk_laddning beskrivs vad laddning är.

Vilken laddning en partikel har bestäms enkelt genom att låta partikeln avlänkas med ett magnetfält med känd riktning.
/Peter E

Nyckelord: standardmodellen [24];

*

Universum-Solen-Planeterna [21242]

Fråga:
Hur vet vi om vi är på samma ställe, om vi är i rymden en dag sen kanske vi åker dit någon månad senar, hur vet vi att vi inte har flyttat oss tex i en låda om jag lägger en penna i en låda sen öppnar den en stund efter så vet ju jag om den har flyttat sig men det kan vi inte veta i rymden. Om vi flyttar oss då flyttar troligtvis allt annat så då kan vi inte mäta med tex månen eller solen. Så med andra ord hur vet vi att vi inte har flyttat oss och om vi kan ta reda på det har vi flyttat oss?
/Anna O, Linnéa skolan, Ljungskile

Svar:
Hej Anna! Det var ingen lätt fråga du ställer. Låt oss hoppas at jag förstått den korrekt: finns absoluta positioner i universum?

Universum är ganska säkert oändligt stort (enligt den kosmologiska standardmodellen, se fråga 18686 ). Om universum är oändligt stort går det inte att definiera absolut position. Vi kan alltså bara definiera relativa positioner i förhållande till jorden, månen, solen, andra stjärnor, pulsarer etc.

Det finns dessutom starkt stöd för den kosmologiska principen (fråga 20517 Kosmologiska_principen ) att universum ser ungefär likadant ut oavsett var man än befinner sig och vart man än tittar.

I ett sådant universum kan vi alltså inte definiera absoluta positioner.

Däremot kan man faktiskt definiera absolut hastighet eftersom den kosmiska bakgrundsstrålningen kan definiera ett koordinatsystem som är unikt, se fråga 16776 .

Se även Absolute_space_and_time#Special_relativity , Location_of_Earth och länk 1 och 2 nedan.
/Peter E

Nyckelord: kosmologi [33];

1 https://www.quora.com/Can-absolute-position-in-the-universe-be-measured-It-seems-as-though-we-only-know-position-relative-to-other-bodies-of-mass
2 https://www.astronomiskungdom.se/2014/03/02/hur-stort-ar-egentligen-universum/

*

Sök efter    

Skriv de ord du vill söka på i sökfältet ovan och klicka på sökknappen. Uteslut ord genom att sätta - (minus) före ordet. Ordgrupper definieras med hjälp av "...". Sökningar är oberoende av stora och små bokstäver.

Exempel:

helium "kalle anka"
Sök på 'helium' och ordgruppen 'kalle anka'
orgelpipa
Sök på 'orgelpipa'
orgel -gitarr
Sök på 'orgel' men inte 'gitarr'

 


sök | söktips | Veckans fråga | alla 'Veckans fråga' | ämnen | dokumentation | ställ en fråga
till diskussionsfora

 

Creative Commons License

Denna sida från NRCF är licensierad under Creative Commons:
Erkännande-Ickekommersiell-Inga bearbetningar
.